Peds Final Exam

अब Quizwiz के साथ अपने होमवर्क और परीक्षाओं को एस करें!

The nurse is caring for an adolescent girl with anorexia nervosa. Which of the following findings would indicate to the nurse that the girl requires hospitalization? Select one: A. Soft, sparse body hair and dry, sallow skin B. Body mass index of 18 C. Weight gain of one-half pound per week D. Food refusal

D. Food refusal Rationale: Food refusal, severe weight loss, unstable vital signs, arrested pubertal development, and the need for enteral nutrition warrant hospitalization. Soft, sparse body hair and dry, sallow skin are signs of anorexia, but do not warrant hospitalization. A weight gain of one-half pound per week indicates progress toward therapeutic goals. A body mass index of 18 is on the low end of the normal range of body mass.

In working with middle to older adolescents with special needs, the nurse teaches the teens when to seek help from a health professional and about the medical insurance process. This nurse is: a. beginning to prepare the teen for transition to adult care b. promoting improved use of the present healthcare resources c. working to reduce the financial burden to the family d. attemping to relieve stressed parents of some responsibility.

a. beginning to prepare the teen for transition to adult care Rationale: Since stranger fear and separation anxiety is a developmental norm around 8 months, attachment is critical to provide security. All the other interventions should be used to promote development, but attachment is essential.

Jamie is an 8-year-old girl admitted to your floor for Cystic fibrosis lung infection. She weighs 46 pounds. What is the maximum safe dose of Ceftazidime for her on a q 8 hour dosing schedule? _____ mg Ceftazidime: Route/Dosage IM, IV (Adults and Children ≥ 12 yr): pneumonia and skin/skin structure infections - 500 mg - 1 g q 8 hr. Bone and joint infections - 2 g q 12 hr. Severe and life-threatening infections - 2 g q 8 hr. Complicated urinary tract infections - 500 mg q 8 hr. Uncomplicated urinary tract infections - 250 mg q 12 hr. Cystic fibrosis lung infections caused by P. aeruginosa - 30 - 50 mg/kg q 8 hr (maximum dose = 6 g/day). IM, IV (Children 1 mo - 12 yr): 33.3-50 mg/kg q 8 hr (maximum dose = 6 g/day). IM, IV (Neonates ≤ 4 wk): 50 mg/kg q 8-12 hr.

1045 mg

A child weighs 6 pounds 4 ounces. Ordered: Ampicillin 400 mg/kg/day in divided doses q 6 hours IV Available: Ampicillin 100 mg/mL What dose will you administer in a single dose? ____ mL

2.84 mL

A mother brings her 8-year-old son for evaluation because of a rash on his lower leg. Which finding would support the suspicion that the child has Lyme disease? Select one: A. Playing in the woods about a week ago B. Complaints of extreme pruritus with visible nits C. High fever occurring about 4 days before the rash D. Rash is papular and vesicular

A. Playing in the woods about a week ago. Rationale: Lyme disease is caused by the bite of an infected tick, with a rash appearing 7 to 14 days after the tick bite. Ticks are commonly found in wooded areas. Therefore, reports of the child playing in the woods about 7 days ago would support the diagnosis of Lyme disease. A papular and vesicular rash is commonly associated with varicella (chickenpox). A high fever for 3 to 5 days before a rash suggests roseola. Extreme pruritus with visible nits would suggest pediculosis.

A nurse is providing an in-service program on child abuse for a group of newly hired nurses. When evaluating the effectiveness of the teaching, the nurse determines a need for additional review when the group identifies which of the following as an indicator of possible child abuse? Select one: A. Sexual behavior that correlates with the child's developmental age B. Frequent changes in history information with visits C. Injuries that are inconsistent with the reported traumatic event D. Consistent delays in seeking treatment for the child's injuries

A. Sexual behavior that correlates with the child's developmental age Rationale: Sexual behavior that correlates with the child's developmental age would be appropriate and not an indicator of child abuse. A delay in seeking medical treatment, a history that changes over time, or a history of trauma that is inconsistent with the observed injury all suggest child abuse.

A child with hypogammaglobulinemia is to receive intravenous immunoglobulin (IVIG). Which of the following would be least appropriate for the nurse to do? Select one: A. Shake the vial after reconstituting it B. Obtain preinfusion vital signs C. Premedicate the child with acetaminophen D. Check serum blood urea nitrogen and creatinine levels

A. Shake the vial after reconstituting it Rationale: Many IVIG products are packed as two vials, one the IVIG powder and one the sterile diluents. Once reconstituted, the IVIG should not be shaken because this leads to foaming and may cause the immunoglobulin protein to degrade. The child can be premedicated with acetaminophen or diphenhydramine. Baseline serum blood urea nitrogen and creatinine should be assessed because acute renal insufficiency may occur as a serious adverse reaction.

The nurse is providing home care instructions for a 13-year-old girl recently diagnosed with systemic lupus erythematosus. Which response by the girl indicates a need for further teaching? Select one: A. "I need an eye examination every year." B. "I need to be careful when it is cold; I should always wear gloves." C. "I need to wear sunscreen in the summer to prevent rashes." D. "I need to eat a healthy diet, exercise, and get plenty of sleep."

C. "I need to wear sunscreen in the summer to prevent rashes." Rationale: The nurse needs to emphasize that the girl should apply sunscreen every day, not just in the summer, to prevent rashes resulting from photosensitivity. A healthy diet, sleep, yearly eye examinations, and protection from cold weather are appropriate measures.

When teaching a class about trisomy 21, the instructor would identify this disorder as due to which of the following? Select one: A. Autosomal dominant inheritance B. X-linked recessive inheritance C. Nondisjunction D. Genomic imprinting

C. Nondisjunction Rationale: Trisomy 21 is an example of a genetic disorder involving an abnormality in chromosomal number due to nondisjunction. X-linked recessive inheritance disorders, such as hemophilia and Duchenne muscular dystrophy, involve altered genes on the X chromosome. Genomic imprinting disorders, such as Prader-Willi syndrome, involve expression of only the maternal or paternal allele, with the other being inactive. Autosomal dominant inheritance disorders, such as neurofibromatosis and achondroplasia, involve a single gene in the heterozygous state that is capable of producing the phenotype, thus overshadowing the normal gene.

A child has just returned from spinal fusion surgery. The nurse should check for signs of: Select one: a. Seizure activity. b. Impaired pupillary response during neurological checks. c. Impaired color, sensitivity, and movement to lower extremities. d. Increased intracranial pressure.

c. Impaired color, sensitivity, and movement to lower extremities. Rationale: When the spinal column is manipulated, there is a risk for impaired color and circulation, sensitivity, and movement to lower extremities.

The nurse is planning care for a school-age child with bacterial meningitis. Which of the following should be included? Select one: a. Avoid giving pain medications that could dull sensorium. b. Measure head circumference to assess developing complications. c. Keep environmental stimuli at a minimum. d. Have the child move her head from side to side at least once every 2 hours.

c. Keep environmental stimuli at a minimum.

The school nurse completes an assessment of a 6-year-old child to determine the services the child will need in the classroom. The child needs respiratory support with oxygen. The child also requires enteral tube feedings and intravenous medication during the school day. With these needs, the school nurse evaluates the child to be: Select one: a. Developmentally delayed. b. Socially withdrawn. c. Medically fragile. d. Mentally handicapped.

c. Medically fragile.

A nurse is assessing an adolescent admitted for a severe ventroperitoneal shunt infection. Which of the following assessment findings would the nurse expect to see? Select one or more: a. Bulging fontanel b. Positive Babinski sign c. Vomiting d. Loss of coordination or balance e. Redness along the shunt tract

c. Vomiting Vomiting is a sign of increased intracranial pressure, which is often present with a shunt infection. WBCs collect in the CSF, and clog the shunt, resulting in shunt malfunction as well as infection. d. Loss of coordination or balance Loss of coordination or balance is a sign of increased intracranial pressure, which may be present with a shunt infection. WBCs collect in the CSF, and clog the shunt, resulting in shunt malfunction as well as infection. e. Redness along the shunt tract Redness along the shunt tract is often present with a shunt infection as a result of the body's response to the infectious agent.

The parents of a 10-year-old boy with cystic fibrosis restrict him from camping, sleepovers with friends, and school field trips. They time his respiratory treatments and log his diet, choosing most foods for him. They have him seen medically between scheduled appointments and call their physician frequently with concerns. The nurse considers these behaviors indicative of: Select one: a. Need for respite care b. Parental denial of the child's condition c. Vulnerable child syndrome d. Parental depression

c. Vulnerable child syndrome Descriptors fit vulnerable child syndrome. Parents "see" the child as more fragile and dependent than he actually is. Resultant protective behaviors on the part of the parents interfere with the boy's ability to grow and develop as normally as possible within the restraints of his illness.

The nurse is caring for a newborn with micrognathia, cleft palate, and glossoptosis. The child is experiencing respiratory distress with severe retractions. What is the priority nursing intervention? Select one: a. Place an NG/OG to decompress the stomach b. Place an oxygen hood on the child c. Begin bag and mask ventilation d. Place the child in a prone position

d. Place the child in a prone position Rationale: This child exhibits signs of Pierre Robin sequence. The accompanying respiratory distress is related to the tongue occluding the airway. Positioning the child prone allows the tongue to fall forward, thereby relieving the obstruction, partially or completely. This is the priority intervention because it is fast, requires no order and will support the child while additional interventions are reviewed.

A 2-year-old has a tonic-clonic seizure while in the hospital crib. The child's jaws are clamped. Which is the most important nursing action at this time? Select one: a. Place a padded tongue blade between the child's jaws. b. Restrain the child to prevent injury. c. Prepare the suction equipment. d. Stay with the child and observe his respiratory status.

d. Stay with the child and observe his respiratory status. Rationale: It is important for the nurse to stay with the child to assess for any changes in the child's respiratory status. Place the child in side-lying position, if possible, to allow secretions to drain. Monitor for adequate oxygenation. The child is at risk for hypoxic injury if the respiratory status is compromised.

A child is diagnosed with Kawasaki disease and is in the acute phase of the disorder. Which of the following would the nurse expect the physician to prescribe? Select all answers that apply. Select one or more: A. Ibuprofen B. Acetaminophen C. Intravenous immunoglobulin D. Aspirin E. Alprostadil

The correct answer is: Intravenous immunoglobulin, Acetaminophen, Aspirin Rationale: In the acute phase, high-dose aspirin in four divided doses daily and a single infusion of intravenous immunoglobulin are used. Acetaminophen is used to reduce fever. Nonsteroidal anti-inflammatory agents such as ibuprofen are avoided while the child is receiving aspirin therapy. Alprostadil is used to temporarily keep the ductus arteriosus patent in infants with ductal-dependent congenital heart defects.

Which of the following would lead the nurse to suspect that an adolescent has bulimia? Select one: A. Body mass index less than 17 B. Nail pitting C. Bradycardia D. Calluses on back of knuckles

D. Calluses on back of knuckles Rationale: The adolescent with bulimia would exhibit calluses on the back of the knuckles and split fingernails and would be of normal weight or slightly overweight. A body mass index of 17, nail pitting, and bradycardia would suggest anorexia.

Are the following symptoms consistent with croup or epiglottitis? A. Cause is usually viral B. Child appears very ill C. Onset is sudden, often at night D. Usually associated with high fever E. Barking cough is a classic sign

A. Cause is usually viral is Croup B. Child appears very ill is Epiglottitis C. Onset is sudden, often at night is Croup, D. Usually associated with high fever is Epiglottitis E. Barking cough is a classic sign is Croup

The nurse is conducting a pain assessment of a 10-year-old boy who has been taking acetaminophen for chronic knee pain. The assessment indicates that the recommended dose is no longer providing adequate relief. What is the appropriate nursing action? Select one: A. Increase the dosage of the acetaminophen. B. Use guided imagery to help his pain. C. Tell the child he is experiencing the ceiling effect. D. Obtain an order for a different medication.

D. Obtain an order for a different medication. Rationale: Increasing the dose of the acetaminophen will not help his pain because he has reached as high a dose of that medication that will work. This is known as the ceiling effect, but explaining that to him will not help his pain. Additionally, acetaminophen is toxic to the liver and increasing the dose could be dangerous. Guided imagery is not the best therapy for his pain, so the physician needs to order a different medication to manage his pain.

The nurse can refer the special needs infant or toddler to an early intervention federal:state program offered at the local level often through: a. The child's home or daycare center. b. food and nutrition programs c. community religious organizations d. the public school system

The child's home or a day care center Rationale: Observing the mother:child interaction during feeding and hygiene activities would disclose lack of knowledge of child care, poor feeding techniques, or inappropriate maternal bonding and interaction as inorganic causes or failure to thrive. The child's lack of interest in or inability to feed would indicate organic causes, as would determining that the child consumed adequate calories for age and finding a history of prematurity or congenital anomaly.

Parents are told by the gentic counselor that they have a 1:4 probability of having a second child with cystic fibrosis (CF). They already have one child who is affected. The parents state their risk is lower now than when they had the previous child. What should the nurse tell the parents about the 1:4 probability? Select one: a. Each pregnancy is an independent event. b. The probability of having another child with CF is twice as likely as it was when they had the first child. c. The probability of having a healthy child is twice as likely with this pregnancy. d. The probability of miscarrying is greater now than with the previous pregnancy.

a. Each pregnancy is an independent event.

The nurse is planning postop care for an infant after a cleft lip repair. The plan should include: Select one: a. Supine or side-lying positioning. b. Prone positioning. c. Avoidance of soft elbow restraints. d. Suctioning with a Yankauer device.

a. Supine or side-lying positioning.

The nurse is assessing a small-for-gestational-age newborn who had an older sibling who died of sudden infant death syndrome. Knowing this, the nurse must include which in the plan of care for the newborn? Select one: a. Encourage the parents to place the infant on his abdomen to sleep. b. Encourage the parents to sleep with the infant for close observation. c. Encourage the parents to place the infant in a crib with a tight-fitting, firm mattress. d. Encourage the parents to place the infant in a crib with a soft mattress with extra blankets.

The correct answer is: Encourage the parents to place the infant in a crib with a tight-fitting, firm mattress.

Knowing that caregivers of a special needs child usually give of themselves in almost unending ways, the nurse will assist the parents by: a. helping them develop workable health-promoting activities for themselves. b. arranging to have the child spend more time in school c. modifying the care plan to focus on only the basic essentials d. stepping in and providing additional direct care

a. helping them develop workable health-promoting activities for themselves. rationale: Only a small percentage of parents routinely take time to promote their own health. Care for the caregiver is essential or the health of the entire family will suffer. In-home-care nurses need to work with the family and not intrude on family function (stepping in to give more direct care). Reducing care to basic essentials is likely not to meet the child's particular needs. Having the child in school longer hours is appropriate only if this serves the child.

The nursing receives report on the following patients: 1. A 3-month-old infant with bronchiolitis who has expiratory wheezing in the upper lobes, decreased lung sounds in the bases and a respiratory rate of 25. 2. A 6-month-old infant with Respiratory Syncitial Virus who has expiratory wheezing a respiratory rate of 71 breaths per minute. 3. A 2-year-old with pneumonia who has expiratory wheezing and a respiratory rate of 55. 4. A 3-year-old with croup who has a barking cough and a respiratory rate is 32. Which patient should the nurse see first?

The answer is patient number 1 Rationale: The 3-month-old is demonstrating signs of respiratory failure and should be assessed first. As the youngest, this child will have the least reserves and is demonstrating diminished lung sounds and a respiratory rate lower than expected for age and condition. The 6-month-old would likely be assessed next due to the high respiratory rate and the potential for additional compromise.

The nurse is preparing to administer ampicillin to a child who weighs 40 kg. The safe dose for children is 50 to 100 mg/kg/day divided in doses administered every 6 hours. What would be the low single safe dose and high single safe dose per day for this child? Select one: A. 500 to 1,000 mg per dose B. 50 to 100 mg per dose C. 1,000 to 5,000 mg per dose D. 100 to 500 mg per dose

A. 500 to 1,000 mg per dose Rationale: To calculate the dosage, the nurse would set up a proportion to calculate the low dose as follows: 50 mg/1 kg = x mg/40 kg; solve for x by cross-multiplying: 1 × x = 50 × 40; x = 2,000 mg divided by 4 doses per day = 500 mg. Then calculate the high safe dose range using the following proportion: 100 mg/1 kg = x mg/40 kg; solve for x by cross-multiplying: 1 × x = 100 × 40; x = 4,000 mg divided by 4 doses per day = 1,000 mg.

An 8-year-old girl with cerebral palsy heard about handicapped horseback riding and is begging to try it. Her mother is frightened of her falling and talks to the nurse. What is the most helpful nursing response?

"The stable has specially trained staff and employs physical therapists to work with the children to provide a specific and safe program." rationale: It is most appropriate to respond to the mother's concern about the perceived danger of the activity. The mother has not asked for information about a stable, the benefits of hippotherapy, or the impact on the daughter's self-esteem.

A child weighs 9 pounds 12 ounces. Ordered: Gentamicin 2.5 mg/kg q 8 hours IV What dose will you administer in a single dose? ____ mg

11.08 mg

A child weighs 11 pounds 12 ounces. Ordered: Gentamicin 2.5 mg/kg q 8 hours IV What dose will you administer in a single dose? ____ mg

13.4 mg

Ordered: IV + PO to equal maintenance Child's weight: 6.3 kg IVF ran at full maintenance all night while the child slept. At 0800, child drinks 46 mL formula. Adjust the rate for the next four hours.

14.8 mL/hour

A child weighs 6 pounds 5 ounces. How many grams is that?

2869 g Rationale: 5/16 = 0.3125 lbs + 6 lbs = 6.3125 lbs 6.3125 lbs/2.2 lbs/kg =2.869318... kg * 1000 g/kg = 2869 g

Ordered: D5 1/2 NS with 20 mEq KCl/L at maintenance. Child weighs 7.2 kg. What is the appropriate hourly rate?

30 mL/hour

A child weighs 9 pounds 3 ounces. How many grams is that?

4176 g Rationale: 9.1875 lbs/ 2.2 lbs/kg = 4.176 kg * 1000 g/kg = 4176 g

Ordered: D5 1/2 NS with 20 mEq KCl/L at maintenance. Child weighs 11.5 kg. What is the appropriate hourly rate?

44.8 mL/hour

A group of nursing students are reviewing information about the endocrine system in infants and children. The students demonstrate understanding of the information when they state which of the following? Select one: A. Endocrine glands begin developing in the third trimester of gestation. B. At birth, the endocrine glands are completely functional. C. Infants have difficulty balancing glucose and electrolytes. D. A child's endocrine system has little effect on growth and development.

C. Infants have difficulty balancing glucose and electrolytes. Rationale: Typically, most endocrine glands begin to develop during the first trimester of gestation, but their development is incomplete at birth. Thus, complete hormonal control is lacking during the early years of life, and the infant cannot appropriately balance fluid concentration, electrolytes, amino acids, glucose, and trace substances.

The nurse is preparing to administer an intramuscular injection to an 8-month-old infant. Which site would the nurse select? Select one: A. Deltoid B. Rectus femoris C. Vastus lateralis D. Dorsogluteal muscle

C. Vastus lateralis Rationale: The preferred injection site in infants is the vastus lateralis muscle. An alternative site is the rectus femoris. The dorsogluteal site is not used in children until the child has been walking for at least 1 year. The deltoid muscle is used as a site in children after the age of 4 or 5 years.

The nurse is preparing to administer a medication to a 5-year-old who weighs 42 pounds. The prescribed single dose is 1 to 2 mg/kg/day. Which of the following is the appropriate dose range for this child? Select one: A. 92 to 185 mg B. 9.5 to 19 mg C. 42 to 84 mg D. 19 to 38 mg

D. 19 to 38 mg Rationale: The nurse should convert the child's weight in pounds to kilograms by dividing the child's weight in pounds by 2.2. (42 pounds divided by 2.2 = 19.09 kg). The nurse would then multiply the child's weight in kilograms by 1 mg for the low end (19.09 kg × 1 mg = 19.09 mg) and then by 2 mg for the high end (19.09 kg × 2 mg = 38.24 mg).

_____ syndrome is the most common sex chromosome abnormality. The karyotype and phenotype are male, but there may be one or more extra X's present. Commonly diagnosed in early adulthood as a result of investigation into the cause of infertility.

Klinefelter

When assessing the adolescent with anorexia, which of the following would the nurse expect to find? Select one: A. Hypertension B. Tachycardia C. Fever D. Murmur

Murmur Rationale: An adolescent with anorexia often exhibits a low body temperature, bradycardia, hypotension, and a murmur resulting from mitral valve prolapse.

The safe dose range for Drug A is 100-300 mg/kg/day. Ordered: 412 mg Q 6 hours. Child's weight: 9 pounds, 8 ounces Is the dose ordered appropriate?

No, 323.85 mg would be the maximum safe dose for Q 6 hour dosing.

_____ is a common cause of infectious diarrhea in children between 6 and 24 months of age. Children with this condition have copious amounts of green, sweet-smelling stool. Two vaccines for this condition are currently licensed for use in infants in the United States.

Rotavirus

A child with growth hormone deficiency is receiving growth hormone. Which of the following would the nurse interpret as indicating effectiveness of this therapy? Select one: A. Rapid weight gain B. Complaints of headaches C. Height increase of 4 inches D. Growth plate closure

C. Height increase of 4 inches Rationale: Effectiveness of growth hormone therapy is indicated by at least a 3- to 5-inch increase in linear growth in the first year of treatment. Rapid weight gain and headaches are adverse reactions of this therapy. The drug is stopped when the epiphyseal growth plates close.

The nurse is planning care for a child with hemolytic uremic syndrome. The child has been anuric and is scheduled for a peritoneal dialysis catheter insertion. While waiting for the catheter insertion to be scheduled, the nurse plans to: Select one: a. Restrict fluids as prescribed. b. Give stool softners to prevent constipation. c. Encourage foods high in potassium. d. Administer analgesics. Feedback

a. Restrict fluids as prescribed.

A 10-month-old with biliary atresia is being discharged after Kasai procedure. Which statement, if made by her parents, indicates that teaching with regard to her prognosis has been understood? Select one: a. "We are happy to be able to stop that special formula and all of those vitamins." b. "We will stop her liver medicine now that she is being discharged." c. "We know that even though surgery is over, she will likely need a liver transplant." d. "We are glad this problem was found so early; now everything will be fine."

c. "We know that even though surgery is over, she will likely need a liver transplant." Rationale: Kasai procedure is palliative, and prognosis is best if performed before 10 weeks of age. Its purpose is to achieve biliary drainage and avoid liver failure. A liver transplant is required in 80 to 90 percent of cases.

T/F: Tympanometry determines the presence of fluid behind the eardrum.

True

The parent of a child with cerebral palsy asks how therapeutic horseback riding might benefit his adolescent. The nurse's response includes: Select all that apply. a. Appetite tends to be stimulated with riding b. Flexibility, balance, and muscle strength tend to improve. c. Sleep problems often resolve. d. Self-esteem and confidence usually get a boost.

b, c, and d rationale: Flexibility, muscle strength, and balance are fostered due to the horse movement as the teen rides. Improvement in these areas would be particularly helpful to the adolescent with cerebral palsy. Improved self-esteem and confidence are also developmentally important as the youth works to establish identity. Improved appetite and resolution of sleep problems are not attributed to therapeutic horseback riding.

The nurse has completed parent education related to treatment for a child with congenital clubfoot. The nurse knows that parents need further teaching when they state: Select one: a. "We'll watch for any swelling of the feet while the casts are on." b. "We're happy this is the only cast our baby will need." c. "We'll keep the casts dry." d. "We're getting a special car seat to accommodate the casts."

b. "We're happy this is the only cast our baby will need."

The nurse is providing family-centered care for a 12-year-old child with pertussis. The child has been ill for 3 weeks. Which of the following treatments are appropriate? Select one or more: a. Oral gentamicin b. Immunization c. Chemoprophylaxis for household members d. IV ampicillin e. Oral azithromycin

The correct answer is: Oral azithromycin, Chemoprophylaxis for household members

A group of students are reviewing information about the anatomic differences in the eyes and ears of a child in comparison to an adult. The students demonstrate a need for additional study when they identify which of the following? Select one: A. Visual acuity develops from birth throughout childhood. B. Binocular vision is usually achieved by 2 months of age. C. The ability to discriminate colors is completed by birth. Correct D. Hearing is completely developed at the time of birth. Feedback

The correct answer is: The ability to discriminate colors is completed by birth. Rationale: The optic nerve is not completely myelinated at birth, so color discrimination is incomplete. Hearing is intact at birth and visual acuity develops from birth throughout childhood. Binocular vision is achieved by 4 months of age.

A lumbar puncture is done on an infant suspected to have meningitis. If the infant has bacterial meningitis, the nurse would expect the cerebral spinal fluid to show what result? Select one: a. An elevated red blood cell count b. A decreased white blood cell count c. Normal glucose d. An elevated white blood cell count

d. An elevated white blood cell count

The nurse works in a pediatric unit. In working with a parent who is suspected of Munchausen syndrome by proxy, it is very important for the nurse to: Select one: a. Confront the parent with concerns of possible abuse. b. Explain to the child that her parent is causing her illness, and that the health team will prevent her from being harmed. c. Try to keep the parent separated from the child as much as possible. d. Carefully document parent—child interactions

d. Carefully document parent—child interactions

The nurse knows that the ____ is the outward characteristics of the individual which are determined by his/her genetic makeup.

phenotype

A nurse is caring for a hospitalized 3 month old infant admitted following a motor vehicle accident. The child is being monitored for increased intracranial pressure. The nurse notes that the anterior fontanel bulges when the infant cries. Based on this assessment finding, which action would the nurse take? Select one: a. Lower the head of the bed b. Have the mother provide comfort measures and reassess. c. Place the infant on NPO status d. Notify the physician immediately

b. Have the mother provide comfort measures and reassess. Rationale: When an infant cries intercranial pressure increases causing the fontanel to bulge. Since crying can occur because of hunger, thirst, pain, the nurse should attempt to decrease the crying by assessing the cause. Notifying the MD first would result in the MD asking the question, "What have you done to decrease the cause of the cry which is increasing the icp?

The nurse assesses a child and finds that the child's pupils are pinpoint. The nurse interprets this finding as indicating which of the following? a. intracranial mass b. brain stem dysfunction c. seizure activity d. brain stem herniation

b. brain stem dysfunction Rationale: Pinpoint pupils are commonly observed in poisonings, brain stem dysfunction, and opiate use. Dilated but reactive pupils are seen after seizures. Fixed and dilated pupils are associated with brain stem herniation. A single dilated but reactive pupil is associated with an intracranial mass.

Which type of genetic test would be used to detect the possibility of Down syndrome? Select one: a. Hemoglobin electrophoresis b. Complete blood count (CBC) c. Chromosomal analysis d. DNA analysis

c. Chromosomal analysis

The nurse is caring for a 2-year-old girl who is wheezing and has difficulty breathing. Which interview question would provide the most useful information related to the symptoms of the child? Select one: A. Inquiring about child safety in the home B. Asking the parents if they smoke in the home C. Asking about the child's diet D. Asking about the temperament of the child

B. Asking the parents if they smoke in the home Rationale: Asking the parents if they smoke in the home would provide the most useful information related to the health condition of the child. If they smoke, the nurse could explain that they are affecting their child's health and urge them to stop for her sake. Asking about the temperament of the child and inquiring about home safety or diet would not reveal any useful information related to the respiratory alteration.

What is the term for the presence of excess fat in the feces (commonly seen in cystic fibrosis)?

steatorrhea

A deficiency in ______ in premature neonates often results in respiratory distress syndrome.

surfactant

The nurse is caring for a hospitalized 8-month-old girl with special healthcare needs. Which intervention would best help this infant grow and develop? a. support parental attachment to the child. b. promote modified gross motor activities c. use play to encourage fine motor skill development d. role model basic care and talk, read, and sing to the child.

a. support parental attachment to the child. Rationale: Attachment interference may already have occurred due to frequent hospitalizations and multiple therapies and treatments. A basic need of this girl is the development of trust in and attachment to her parents. Since stranger fear and separation anxiety is a developmental norm around 8 months, attachment is critical to provide security. All the other interventions should be used to promote development, but attachment is essential.

The nurse is teaching the parents of a newly diagnosed cystic fibrosis client how to administer the pancreatic enzymes. The nurse will advise the parents to administer the enzymes: Select one: a. b.i.d. (twice a day) b. With meals and large snacks. c. q.i.d. (four times a day) d. Every 6 hours around the clock.

b. With meals and large snacks. Rationale: Pancreatic enzymes are administered with meals and large snacks. A scheduled time would not be appropriate because the enzymes are used to assist in digestion of nutrients.

The nurse is caring for an infant born prematurely. Which intervention is performed based on chronological age? a. assessment of growth & development b. administration of immunizations c. use of an infant formula for full-term babies d. provision of anticipatory guidance for parents

b. administration of immunizations rationale: Infants born prematurely should receive all immunizations based on their chronological age. The other interventions should be based on the infants' corrected or adjusted age.

The nurse caring for a hospitalized child with failure to thrive (FTT) will focus first on: a. forming a positive relationship with the child b. assisting the child to attain adequate nutrition to demonstrate weight gain c. providing appropriate developmental stimulation d. determining the quality of the parent-child relationship

b. assisting the child to attain adequate nutrition to demonstrate weight gain Attaining nutrition to promote weight gain is the primary focus. Special feeding situations and methods may be needed such as desensitizing the child to certain food textures or beginning enteral feedings. All the other options are important in helping the child with FTT but are not the initial focus.

A nurse is teaching the parents of an infant with congenital adrenal hyperplasia about the signs and symptoms of adrenal crisis. The nurse determines that the teaching was successful when the parents identify which of the following? Select one: A. Persistent vomiting B. Fluid overload C. Constipation D. Bradycardia

A. Persistent vomiting Rationale: Signs and symptoms of acute adrenal crisis include persistent vomiting, dehydration, hyponatremia, hyperkalemia, hypotension, tachycardia, and shock.

The nurse working in the emergency room monitors the admission of children. Statistically, for which one of the following disorders would children younger than 5 years most commonly be admitted? Select one: A. Respiratory disorders B. Mental health problems C. Injuries D. Gastrointestinal disorders Feedback

A. Respiratory disorders According to Child Health USA 2008-2009, diseases of the respiratory system account for the majority of hospitalizations in children younger than 5 years of age, while diseases of the respiratory system, mental health problems, injuries, and gastrointestinal disorders lead to more hospitalizations in older children.

Ordered: D10W at maintenance. Child weighs 3562 g. What is the appropriate hourly rate?

14.8 mL/hour Rationale: 3.562 kg = 356.2 ml/24 hours. Hourly rate: 14.8 ml/hr

A child with attention deficit/hyperactivity disorder is prescribed long-acting methylphenidate. Which of the following would the nurse include when teaching the child and his parents about this drug? Select one: A. "Take this drug every day in the morning when you wake up." B. "Some increase in appetite may occur, so watch how much you eat." C. "This drug may cause drowsiness, so be careful when doing things." D. "Give the drug three times a day: morning, midday, and after school."

A. "Take this drug every day in the morning when you wake up." Rationale: Long-acting methylphenidate is administered once daily in the morning, whereas the other forms are given three times a day. The drug typically causes difficulty sleeping and decreased appetite.

A nurse is conducting a screening program for autism in infants and children. Which of the following would the nurse identify as a warning sign? Select one: A. Inability to say a single word by 16 months B. Lack of babbling by 6 months C. Lack of gestures by 8 months D. Inability to use two words by 18 months

A. Inability to say a single word by 16 months Rationale: Warning signs of autism include no babbling by 12 months, no pointing or using gestures by 12 months, no single words by 16 months, no two-word utterances by 24 months, and loss of language or social skills at any age.

An Asian family brings their special needs school-ager in for a routine visit. What use of adjuvant therapies would it be particularly wise for the nurse to explore? a. music therapy b. pet therapy c. herbal remedies d. massage

c. herbal remedies Herbal remedies are commonly used in Eastern medicine. These can be helpful, neutral, or detrimental. Knowledge of their use is important so the impact on the child can be evaluated. The other therapies are less likely to have a negative effect and can be quite helpful. Knowledge of all therapies and their integration into a total care package remains essential.

A child with spastic cerebral palsy is to receive botulin toxin. The nurse prepares the child for administration of this drug by which route? Select one: A. Subcutaneous injection B. Intramuscular injection C. Oral D. Intravenous infusion

B. Intramuscular injection Rationale: Botulin toxin is administered by injection into the muscle. It may cause dry mouth. It is not administered orally, by subcutaneous injection, or by intravenous infusion.

Which of the following are appropriate interventions in the management of severe brain injury? Select one or more: a. Mannitol administration b. Seizure prevention/treatment c. Decreasing environmental stimulation d. Invasive ICP monitoring e. Assisting with electromyography

The correct answer is: Mannitol administration, Invasive ICP monitoring, Seizure prevention/treatment, Decreasing environmental stimulation

The nurse is examining a 2-year-old child who was adopted from Guatemala. Which of the following would be a priority screening for this child? Select one: A. Screening for abuse B. Screening for childhood illnesses C. Screening for infectious diseases D. Screening for congenital defects

C. Screening for infectious diseases Rationale: Although all the screenings are important, health supervision of the internationally adopted child must include comprehensive screening for infectious disease. In 2008, approximately 19,600 children were adopted from countries outside the United States, many from areas with a high prevalence of infectious diseases (Intercountry Adoption, Office of Children's Issues, U.S. Department of State, 2010a, 2010b). Guatemala, China, and Russia supplied about half of all international adoptees in 2008, followed by Ethiopia, South Korea, and Vietnam. Proper screening is important not only to the child's health but also to the adopting family and the larger community.

A 15-year-old boy comes to the emergency department accompanied by his parents. The boy reports an abrupt onset of sudden pain on the right side of his scrotum. When asked to rate his pain on a scale of 1 to 10, with 10 being the most severe, the boy states, "It's a 12." Further assessment reveals a blue-black swelling on the affected side. The nurse suspects testicular torsion and immediately notifies the physician based on the understanding of which of the following? Select one: A. Renal failure is imminent. B. The boy is at risk for sepsis. C. Intravenous antibiotics need to be initiated. D. The condition is a surgical emergency.

The correct answer is: The condition is a surgical emergency. Rationale: Testicular torsion is a surgical emergency that necessitates immediate surgical correction to prevent testicular necrosis and possible gangrene. There is no infection with testicular torsion, intravenous antibiotics are not used to treat this condition, and renal failure is not imminent.

Ordered: IV + PO to equal maintenance Child's weight: 8.5 kg IVF ran at full maintenance all night while the child slept. At 0800, child drinks 53 mL apple juice. Adjust the rate for the next four hours.

22 mL/hour Rationale: 8.5 kg * 100ml/day = 850 ml/day = maintenance = 35.4 ml/hour 4 hours worth = 141.6 Minus the 53 the child took po = 88.6 to be made up with IVF/4 hours = 22.15 ml/hr

Ordered: D5 1/2 NS with 20 mEq KCl/L at maintenance. Child weighs 18 kg. What is the appropriate hourly rate?

58.3 mL/hour

Ordered: D5 1/2 NS with 20 mEq KCl/L at maintenance. Child weighs 18 kg. What is the appropriate hourly rate?

58.3 ml/hr

The nurse is counseling the parents of a 9-year-old boy who is receiving morphine for postoperative pain. Which of the following statements from the nurse accurately reflects the pain experience in children? Select one: A. "It is very rare that children become addicted to narcotics." B. "Your child will learn to adapt to the pain he is experiencing." C. "Your child will experience more adverse effects to narcotics than adults." D. "You can expect that your child will tell you when he is experiencing pain."

A. "It is very rare that children become addicted to narcotics." Rationale: Addiction to narcotics when used in children is very rare. Often children deny pain to avoid a painful situation or procedure, embarrassment, or loss of control. Repeated exposure to pain or painful procedures can result in an increase in behavioral manifestations. The risk of adverse effects of narcotic analgesics is the same for children as for adults.

The nurse is assessing a 5-year-old girl who is anxious, has a high fever, speaks in a whisper, and sits up with her neck thrust forward. Based on these findings, which of the following would be least appropriate for the nurse to perform? Select one: A. Auscultating for lung sounds B. Providing 100% oxygen C. Having the child sit forward D. Visualizing the throat

D. Visualizing the throat The child is exhibiting signs and symptoms of epiglottis, which can be life-threatening. Under no circumstances should the nurse attempt to visualize the throat. Reflex laryngospasm may occur, precipitating immediate airway occlusion. Providing 100% oxygen in the least invasive manner that is most acceptable to the child is a sound intervention, as is allowing the child to assume a position of sitting forward with the neck extended. Auscultation would reveal breath sounds consistent with an obstructed airway.

T/F: Recurrent otitis media does not affect language development in infants and toddlers.

False

After teaching the parents of a child diagnosed with celiac disease about nutrition, the nurse determines that the teaching was effective when the parents identify which foods as appropriate for their child? Select all answers that apply. Select one or more: A. Carbonated drinks B. Peanut butter C. Jelly D. Shellfish E. Flavored yogurt F. Wheat germ

The correct answer is: Peanut butter, Carbonated drinks, Shellfish, Jelly Rationale: Foods allowed in a gluten-free diet include peanut butter, carbonated drinks, shellfish, and jelly. Wheat germ and flavored yogurt should be avoided.

Which of the following should be included when educating patients/parents of early warning signs of an asthma attack? (Select all that apply.) Select one or more: a. Diaphoresis b. Extra effort to breathe c. Restlessness while trying to sleep d. Light wheezing e. Abdominal pain from severe coughing

The correct answer is: Restlessness while trying to sleep, Extra effort to breathe, Light wheezing

A parent reports that her 5-year-old child, who has had all recommended immunizations, had a mild fever 1 week ago, and now has bright red cheeks and a lacy red rash on the trunk and arms. The nurse recognizes that this child might have: Select one: a. German measles (rubella). b. Fifth disease (Erythema infectiosum). c. Chickenpox (varicella). d. Roseola (Exanthem subitum). Feedback

b. Fifth disease (Erythema infectiosum).

An infant has been born with an esophageal atresia and tracheoesophageal fistula. What is a priority preop nursing diagnosis? Select one: a. Ineffective tissue perfusion: gastrointestinal related to decreased circulation b. Acute pain related to esophageal defect c. Risk for aspiration related to regurgitation d. Ineffective infant feeding pattern related to uncoordinated suck and swallow

c. Risk for aspiration related to regurgitation

A 28-day-old infant with a fever is admitted to rule out sepsis. Blood, urine, and CSF cultures are drawn and the child is placed on IV antibiotics. The following day the urine culture comes back positive. The nurse knows that a common cause of UTI in a child this age is: Select one: a. Nephrotic syndrome b. Ureteral pelvic junction obstruction c. Vesicoureteral reflux d. Hypospadias

d. Hypospadias Rationale: Hypospadias is a condition in which the urethral opening is on the ventral surface of the penis.70% of young children diagnosed with UTI have vesicoureteral reflux, a condition in which urine from the bladder flows back up the ureters. This reflux of urine occurs during bladder contraction with voiding.

A 7 year old child is seen in a clinic, and the primary health care provider documents a diagnoses of nocturnal enuresis. When the mother asks the nurse about the diagnoses, the nurses responds knowing that nocturnal enuresis Select one: a. Is caused by psychiatric problems b. Requires surgical intervention to improve the problem c. Does not respond to treatment d. Is common, and most children will outgrow the problem without therpeutic intervention

d. Is common, and most children will outgrow the problem without therpeutic intervention

A group of nursing students are reviewing information about the variations in respiratory anatomy and physiology in children in comparison to adults. The students demonstrate understanding of the information when they identify which of the following? Select one: A. Children develop hypoxemia more rapidly than adults do. B. Children's bronchi are wider in diameter than those of an adult. C. Children's demand for oxygen is lower than that of adults. D. An increase in oxygen saturation leads to a much larger decrease in pO2.

A. Children develop hypoxemia more rapidly than adults do. Rationale: Children develop hypoxemia more rapidly than adults do because they have a significantly higher metabolic rate and faster resting respiratory rates than adults do, which leads to a higher demand for oxygen. A smaller decrease in oxygen saturation reflects a disproportionately much larger decrease in pO2. The bronchi in children are narrower than in adults, placing them at higher risk for lower airway obstruction.

The nurse working with children in a hospital setting notes that they are being discharged earlier and earlier. Which of the following is a primary reason for this trend? Select one: A. Cost containment B. Increased funding for home care C. Nursing shortages D. National health care initiatives

A. Cost containment Rationale: Over the past century changes in health care, such as strained health care funding, shorter hospital stays, and cost containment, have led to a shift in responsibilities of care for children from the hospital to homes and communities. Nursing shortages influence the delivery of health care. National health care initiatives may or may not affect earlier discharge to home health care.

A child with diabetes insipidus is being treated with vasopressin. The nurse would assess the child closely for signs and symptoms of which of the following? Select one: A. Syndrome of inappropriate antidiuretic hormone (SIADH) B. Cushing syndrome C. Thyroid storm D. Vitamin D toxicity

A. Syndrome of inappropriate antidiuretic hormone (SIADH) Rationale: SIADH, although rare in children, is a potential complication of excessive administration of vasopressin. Thyroid storm may result from overadministration of levothyroxine (thyroid hormone replacement). Cushing syndrome is associated with corticosteroid use. Vitamin D toxicity may result from the use of vitamin D as treatment of hypoparathyroidism.

Supportive care for the family of an infant with sudden infant syndrome (SIDS) includes: Select one: a. Sheltering the parents from grief by not giving them any personal items of the infant's, such as footprints. b. Interviewing the parents to determine the cause of the SIDS incident. c. Advising the parents that an autopsy is not necessary. d. Allowing the parents to hold, touch, and rock the infant.

d. Allowing the parents to hold, touch, and rock the infant. Rationale: The parents should be allowed to hold, touch, and rock the infant, giving them a chance to say good-bye to their baby.

What intervention should nurses choose to assist the family with a special needs child to function in as healthy a manner as possible? a. Encourage care of the child in a setting outside the family's home. b. Focus on preventing denial of the child's condition by the mother and father. c. Explain that the situation will become less confusing and easier to handle with time. d. Involve both mother and father in explanations and teaching with special attention to keeping the father connected and informed.

d. Involve both mother and father in explanations and teaching with special attention to keeping the father connected and informed. Rationale: Keeping both the mother and father care capable and well informed is essential. Usually mothers carry the larger burden of care and have the greater contact with the physicians or nurse practitioners. Therefore, nurses' efforts are needed to keep fathers vital and involved. Denial is not always a negative. Some denial can be a source of hope. Most families are more comfortable keeping a special needs child within the family unit, thereby understanding that child well and finding enjoyment in the child's changes and accomplishments. Time that allows for adjusting to the situation, mastering care required, and developing a family rhythm will usually make the situation easier to handle, but for the nurse to explain this may sound hollow and unsympathetic.

As a result of seizure activity, a computed tomography (CT) scan was performed and showed that an 18-month-old child has intracranial arteriovenous malformation. When developing the child's plan of care, the nurse would expect to implement actions to prevent which of the following? Select one: A. Hemorrhagic stroke B. Developmental disabilities C. Respiratory paralysis D. Drug interactions

A. Hemorrhagic stroke Rationale: Intracranial hemorrhage or hemorrhagic stroke is a risk for children with intracranial arteriovenous malformation. Drug interactions are a risk for children who are treated with combinations of anticonvulsants for epilepsy. Children with hydrocephalus are at an increased risk for developmental disabilities. Respiratory paralysis is a risk of botulism that typically affects infants younger than 6 months of age.

After teaching a class about inborn errors of metabolism, the instructor determines that additional teaching is needed when the class identifies which of the following as an example of an inborn error of metabolism? Select one: A. Maple syrup urine disease B. Achondroplasia C. Tay-Sachs disease D. Galactosemia

Achondroplasia Rationale: Achondroplasia is an autosomal dominant genetic disorder, not an inborn error of metabolism. Galactosemia, maple syrup urine disease, and Tay-Sachs are considered inborn errors of metabolism.

In assessing a premature child, the nurse will discontinue using the corrected age when the child reaches _________________ years of age.

Age 3 years is true for most assessments. However, the Denver II discontinues adjusting the calculated age at 2 years.

A 4-year-old is brought to the emergency department with a burn. Which of the following would alert the nurse to the possibility of child abuse? Select one: A. The burn area appears asymmetric and nonuniform. B. Clear delineations are noted between burned and nonburned skin areas. C. Burn assessment correlates with mother's report of contact with a portable heater. D. Parents state that the injury occurred approximately 15 to 20 minutes ago.

B. Clear delineations are noted between burned and nonburned skin areas. Rationale: Suggested signs of a burn resulting from possible child abuse include a uniform appearance of the burn with clear delineations of burned and nonburned areas. Abuse would also be suspected if the report of the injury was inconsistent with burn injury or there was a delay in seeking treatment. An asymmetric nonuniform burn often correlates with a splatter-type burn resulting from the child pulling a source of hot fluid onto himself or herself.

The nurse is reviewing the laboratory test results of a child with nephrotic syndrome. Which of the following would the nurse least likely expect to find? Select one: A. Hypoproteinemia B. Decreased blood urea nitrogen (BUN) C. Hyperlipidemia D. Hypoalbuminemia

B. Decreased blood urea nitrogen (BUN) Rationale: With nephrotic syndrome, proteinuria, hyperlipidemia, decreased serum protein levels (hypoproteinemia), and decreased serum albumin levels (hypoalbuminemia) are present. BUN typically becomes elevated.

After teaching a class about the differences in the skin of infants and adults, the nurse determines that additional teaching is necessary when the class states which of the following? Select one: A. "An infant has less subcutaneous fat, which places the infant at a higher risk for heat loss." B. "The infant's epidermis is loosely connected to the dermis, increasing the risk for breakdown." C. "An infant's skin is thinner than an adult's, so substances placed on the skin are absorbed more readily." D. "The infant has a lower risk for damage from ultraviolet radiation because the skin is more pigmented."

D. "The infant has a lower risk for damage from ultraviolet radiation because the skin is more pigmented." Rationale: Infants have less pigmentation in their skin, placing them at increased risk for skin damage from ultraviolet radiation. The infant's skin is thinner, the epidermis is loosely connected, and there is less subcutaneous fat.

A child with depression is prescribed fluoxetine. The nurse identifies this as belonging to which class of drugs? Select one: A. Atypical antidepressant B. Tricyclic antidepressant C. Psychostimulant D. Selective serotonin reuptake inhibitor

D. Selective serotonin reuptake inhibitor Rationale: Fluoxetine (Prozac) is a selective serotonin reuptake inhibitor. Trazodone is an atypical antidepressant; amitriptyline, desipramine, imipramine, and nortriptyline are tricyclic antidepressants. Methylphenidate and the amphetamines are psychostimulants.

An infant with Tetralogy of Fallot is having a hypercyanotic episode. Which of the following nursing interventions should the nurse implement? Select all that apply. Select one or more: a. Administer oxygen. b. Administer demerol as ordered PRN. c. Administer diphenhydramamine (Benadryl) as ordered PRN. d. Place the child in knee-chest position. e. Draw blood for a serum hemoglobin.

The correct answer is: Administer oxygen., Place the child in knee-chest position.

A community health nurse has been asked to talk about preventing accidental drowning at a child safety day for parents of toddlers. Which of the following would the nurse emphasize? Select one or more: A. Children this age are at increased risk for accidental poisoning due to their natural curiosity and increased mobility. B. Water wings or "floaties" are not a substitute for personal flotation devices or adult supervision. C. Children this age should ride in a rear facing car seat until 2 years of age. D. Children this age are most likely to drown in artificial pools, toilets, and liquid-filled buckets.

The correct answer is: Children this age are most likely to drown in artificial pools, toilets, and liquid-filled buckets., Water wings or "floaties" are not a substitute for personal flotation devices or adult supervision.

A 15-year-old adolescent is brought to the emergency department by his parents. The adolescent is febrile with chills that started suddenly. He states, "I had a sinus infection and sore throat a couple of days ago." The nurse suspects bacterial meningitis based on which of the following? Select all answers that apply. Select one or more: A. Complaints of stiff neck B. Photophobia C. Vomiting D. Negative Brudzinski sign E. Absent headache

The correct answer is: Complaints of stiff neck, Photophobia, Vomiting Rationale: In addition to the adolescent's complaints and history, other findings suggesting bacterial meningitis include complaints of a stiff neck, photophobia, headache, positive Brudzinski sign, and vomiting.

A nurse is preparing a presentation for a group of parents with children diagnosed with diabetes type 1. The children are all adolescents. Which of the following issues would the nurse need to address? Select all that apply. Select one or more: A. Self-monitoring of blood glucose levels B. Feelings of being different C. Body image conflicts D. Struggle for independence E. Deficient decision-making skills

The correct answer is: Deficient decision-making skills, Body image conflicts, Struggle for independence Rationale: Adolescents are undergoing rapid physical, emotional, and cognitive growth. Working toward a separate identity from parents and the demands of diabetic care can hinder this. This struggle for independence can lead to nonadherence of the diabetic care regimen. Conflicts develop with self-management, body image, and peer group acceptance. Teens may acquire the skills to perform tasks related to diabetic care but may lack decision-making skills needed to adjust treatment plan. Teens do not always foresee the consequences of their activities. Self-monitoring of blood glucose levels and feelings of being different are issues common to school-age children.

A nursing student is reviewing information about primary immunodeficiencies. The student demonstrates understanding of the material by identifying which of the following as affecting only males? Select all answers that apply. Select one or more: A. Selective IgA deficiency B. IgG subclass deficiency C. Severe combined immune deficiency D. X-linked hyper-IgM syndrome E. Wiskott-Aldrich syndrome F. X-linked agammaglobulinemia

The correct answer is: X-linked agammaglobulinemia, Wiskott-Aldrich syndrome, X-linked hyper-IgM syndrome

The nurse knows that a ___ can result from congenital malrotation of the bowel. This condition results in obstruction of the bloodflow to the affected area and is considered a surgical emergency.

Volvulus

A child is admitted to the hospital unit with physical injuries. The nurse is taking the child's history. Which statement by the parent is most suspicious for abuse? Select one: a. "The baby's 18-month-old brother was trying to pull the baby out of the crib and dropped the baby on the floor." b. "I feel so bad. I was holding the baby in one arm and some towels in the other and did not see my toddler's toys on the floor. I tripped, and when I fell, the baby slipped out of my arms." c. "I placed the baby in the infant swing. His 6-year-old brother was running through the house and tripped over the swing, causing it to fall." d. "I did not realize that my baby was able to roll over yet, and I was just gone a minute to check on dinner when the baby rolled off of the couch and onto our tile floor."

a. "The baby's 18-month-old brother was trying to pull the baby out of the crib and dropped the baby on the floor." The statement "The baby's 18-month-old brother was trying to pull the baby out of the crib and dropped the baby on the floor" is the most suspicious for abuse because developmentally, it would be very difficult for an 18-month-old child to pull an infant out of a crib.

A nurse is doing a postop assessment on an infant who has just had a ventroperitoneal shunt placed for hydrocephalus. Which assessment would indicate a malfunction in the shunt? Select one: a. Bulging fontanelle b. Negative Brudzinski sign c. Incisional pain d. Movement of all extremities

a. Bulging fontanelle

A child with a myelomeningocele corrected at birth is now 5 years old. What is a priority nursing diagnosis for a child with corrected spina bifida at this age? Select one: a. Risk for altered urinary elimination b. Risk for altered comfort c. Risk for infection d. Risk for impaired tissue perfusion-cranial

a. Risk for altered urinary elimination Children with spina bifida are prone to neurogenic bladder. The bladder may not empty completely with voiding, resulting in urinary retention.

An 8-year-old boy with Duchenne muscular dystrophy is being seen in the clinic for a routine health visit. An appropriate nursing diagnosis for this client would be: Select one: a. Risk for injury related to muscle weakness. b. Risk for impaired skin integrity related to paresthesia to lower extremities. c. Risk for infection related to altered immune system. d. Risk for altered comfort related to effects of muscular dystrophy disease.

a. Risk for injury related to muscle weakness.

A child is admitted with infective endocarditis. Which of the following is the nurse's priority action? Select one: a. Start an intravenous line. b. Place the child on seizure precautions. c. Place the child in contact isolation. d. Assist with a lumbar puncture.

a. Start an intravenous line.

The nurse has admitted a child with a cyanotic heart defect. Which initial lab result would the nurse expect to find? Select one: a. Low platelet count b. Low hematocrit c. High hemoglobin and hematocrit d. High white blood cell count

c. High hemoglobin and hematocrit The child's bone marrow responds to chronic hypoxemia by producing more red blood cells to increase the amount of hemoglobin available to carry oxygen to the tissues. This occurs in cases of cyanotic heart defects. Therefore, the hematocrit would not be low, the white blood cell count would not be high (unless an infection were present), and the platelets would be normal.

A child has sustained a basilar skull fracture. For which complication should the nurse assess? Select one: a. Transient confusion b. Periorbital ecchymosis c. Headache d. Cerebral spinal fluid leakage from the nose or ears

d. Cerebral spinal fluid leakage from the nose or ears. CSF leakage indicates a tear in the dural membrane, resulting in a direct opening from the environment into the brain. This can place the child at higher risk for infection. Surgical repair is performed if it persists after 7 - 10 days. Rationale: A fracture of the bones that form the base (floor) of the skull and results from severe blunt head trauma of significant force. A basilar skull fracture commonly connects to the sinus cavities. This connection may allow fluid or air entry into the inside of the skull and may cause infection. The key word is complication. While periorbital ecchymosis and CSF are both signs of a basilar skull fracture only the CSF leak is a complication because it involves an opening from the brain to the outside which provides a route for serious infection, primarily meningitis. A fracture of the bones that form the base (floor) of the skull and results from severe blunt head trauma of significant force. A basilar skull fracture commonly connects to the sinus cavities. This connection may allow fluid or air entry into the inside of the skull and may cause infection.

A 45-day-old infant weighing 4.2 kg is admitted with suspected Herpes encephalitis. He has an IV in place and is receiving compatible maintenance IV fluid and his first dose of Acyclovir 113 mg. The drug book information on dosing is as follows: Acyclovir Route/dosage Herpes Simplex Encephalitis IV (Adults): 10 mg/kg q 8 hr for 14-21 days. IV (Children 3 mo - 12 yr): 10 mg/kg q 8 hr for 14-21 days. IV (Children birth - 3 mo): 20 mg/kg q 8 hr for 14-21 days. IV (Neonates, premature): 10 mg/kg q 12 hr for 14-21 days. What is the priority nursing intervention at this point? Select one: a. Complete a full assessment, including evaluation of neurologic status. b. Increase the maintenance IV fluid to dilute the acyclovir infusion. c. Assess IV site for any sign of infiltration or phlebitis. d. Stop the infusion & notify the physician

d. Stop the infusion & notify the physician Rationale: The maximum recommended dose for a child this weight is 84 mg. Renal failure, seizures, and other life-threatening sequelae may occur with overdose. The infusion should be stopped until the conflict is resolved.

What is the medical term for nosebleed?

epistaxis

A _______ is a congenital malformation in which the intra-abdominal contents herniate through the umbilical cord.

omphalocele

The nurse is assessing a 4-year-old boy whose mother was 40 years old when he was born. Which of the following findings suggests this child has a genetic disorder? Select one: A. Inspection reveals low-set ears with lobe creases. B. Inquiry determines the child had feeding problems. C. Observation shows nasal congestion and excess mucus. D. Auscultation reveals the presence of wheezing.

A. Inspection reveals low-set ears with lobe creases. Rationale: Low-set ears are associated with numerous genetic dysmorphisms. Additionally, the mother's age during pregnancy is a risk factor for genetic disorders. Feeding problems could have been due to low birthweight, prematurity, or a variety of other reasons. The nasal congestion may be a cold. The wheezing could be bronchiolitis or asthma.

The nurse is taking a health history of a child with suspected acute poststreptococcal glomerulonephritis. Which of the following responses would alert the nurse to a confirmed risk factor for this condition? Select one: A. "My child has not been sick at all." B. "My child is just 18 months old." C. "He just got over a head cold with laryngitis." D. "She has been very healthy up to now."

C. "He just got over a head cold with laryngitis." Rationale: Known risk factors include a recent episode of pharyngitis or other streptococcal infection, age older than 2 years, and male sex.

The nurse is caring for children who are receiving IV therapy in the hospital setting. For which of the following children would a central venous device be indicated? Select one: A. A child who is receiving antibiotics for 21 days B. A child who is receiving a one-time dose of a medication C. A child who is receiving an IV push D. A child who is receiving IV fluids for dehydration

A. A child who is receiving antibiotics for 21 days Rationale: Although central venous access devices can be used short term, the majority are used for moderate- to long-term therapy, such as chemotherapy or other longer term medications. Central venous access devices are indicated when the child lacks suitable peripheral access, requires IV fluid or medication for more than 3 to 5 days, or is to receive specific treatments, such as the administration of highly concentrated solutions or irritating drugs that require rapid dilution. Peripheral IV devices are used for most other IV therapies.

The nurse is monitoring a child who has received epidural analgesia with morphine. The nurse is careful to monitor for which of the following adverse effects of the medication? Select one: A. Respiratory depression B. Epidural hematoma C. Spinal headache D. Arachnoiditis

A. Respiratory depression Rationale: The nurse needs to monitor for signs of respiratory depression, a potential adverse effect of the opioid medication. Epidural hematoma, arachnoiditis, and spinal headache are potential adverse effects of the insertion of the epidural catheter.

The nurse is caring for a newborn infant who has Down syndrome. Which of the following nursing actions reflects the nurse's use of the ethical principle of nonmaleficence? Select one: A. The nurse fairly allocates resources for caring for newborns in a facility. B. The nurse provides safe, competent nursing care to avoid harming the infant. C. The nurse involves the parents in making health care decisions for their child. D. The nurse speaks truthfully to the parents regarding their child's prognosis.

B. The nurse provides safe, competent nursing care to avoid harming the infant. Rationale: Ethics includes the basic principles of autonomy, beneficence, nonmaleficence, justice, veracity, and fidelity. Nonmaleficence means avoiding causing harm, intentionally or unintentionally. One example is providing safe, competent nursing care. Speaking truthfully to the parents is an example of veracity. Generally, parents have the autonomy to make health care decisions for their child. Justice refers to acting fairly, and also involves allocating resources fairly.

The nurse knows that the emancipated minor is considered to have the legal capacity of an adult and may make his or her own health care decisions. Which of the following children would potentially be considered an emancipated minor? Select one: A. A minor who is pregnant B. A child older than 13 years of age who asks for emancipation C. A minor who puts his or her medical decisions in writing D. A minor with financial independence who is living with his parents Feedback

A. A minor who is pregnant Rationale: Emancipation may be considered in any of the following situations, depending on the state's laws: membership in a branch of the armed services, marriage, court-determined emancipation, financial independence and living apart from parents, college attendance, pregnancy, mother younger than 18 years of age, and a runaway.

A nurse develops a plan of care for a child that includes patching the eye. This plan of care would be most appropriate for which condition? Select one: A. Astigmatism B. Amblyopia C. Hyperopia D. Myopia

B. Amblyopia Rationale: Eye patching is used for amblyopia or any condition that results in one eye being weaker than the other. Corrective lenses would be appropriate for astigmatism, hyperopia, and myopia.

A child is brought to the emergency department by his parents because he suddenly developed a barking cough. Further assessment leads the nurse to suspect that the child is experiencing croup. Which of the following would the nurse have most likely assessed? Select one: A. High fever B. Toxic appearance C. Dysphagia D. Inspiratory stridor

D. Inspiratory stridor Rationale: A child with croup typically develops a barking-like cough often at night. This may be accompanied by inspiratory stridor and suprasternal retractions. Temperature may be normal or slightly elevated. A high fever, dysphagia, and toxic appearance are associated with epiglottitis.

A child weighs 12 pounds 6 ounces. Ordered: Gentamicin 2.5 mg/kg q 8 hours IV Available: Gentamicin 8 mg/mL What dose will you administer in a single dose? ____ mL

The correct answer is: 1.76

A child weighs 5 pounds 12 ounces. How many grams is that?

The correct answer is: 2614 Rationale: 5.75 lbs/2.2 lbs/kg = 2.6136 kg * 1000 g/kg = 2613.6 g -> 2614 g is reasonable.

The charge nurse on a hospital unit is developing plans of care related to separation anxiety. The charge nurse recognizes that which hospitalized child is at greatest risk to experience separation anxiety when parents cannot stay? Select one: a. 18-month-old b. 3-year-old c. 4-year-old d. 6-month-old

a. 18-month-old While all of these children can experience separation anxiety, the young toddler is at greatest risk. Toddlers are the group at greatest risk for a stressful experience when hospitalized. Separation from parents increases this risk greatly.

A nurse receives report on a 3-year-old child with a diagnosis of asthma exacerbation. The offgoing nurse reports that the child has been agitated and diaphoretic. She was very restless and would not lie down. Her oxygen need has increased in the past 24 hours. The child received Albuterol q 2 hours this shift. When the new nurse enters the room, the child is falling asleep in her mother's arms. Mother verbalizes relief that the child is finally able to sleep. On assessment, the child is no longer wheezing. She does not wake with assessment or VS. The nurse does which of the following? Select one: a. Contacts the attending physician to request a chest x-ray b. Tries to wake the child and administers albuterol c. Contacts the rapid response team for immediate assistance d. Lets the child rest for the time and reassesses in 2 hours

c. Contacts the rapid response team for immediate assistance Rationale: Agitation, diaphoresis, restlessness and the inability to lie down are all signs of respiratory failure. The child has been experiencing these symptoms for an unknown period of time. Absence of breath sounds or wheezing is disturbing as it indicates that air flow is so diminished that it cannot even cause wheezing. When accompanied by confusion or drowsiness, respiratory arrest is imminent.

A child has sustained a traumatic brain injury, and is being monitored in the pediatric intensive care unit. The nurse is using the Glasgow Coma Scale to assess the child. Which assessments will be included? Select all that apply. Select one or more: a. Verbal response b. Head circumference c. Eye opening d. Pulse oximetry e. Motor response

The correct answer is: Verbal response, Eye opening, Motor response

Which exercise would the nurse suggest as most helpful to maintain mobility in a child with juvenile idiopathic arthritis? Select one: A. Swimming B. Playing basketball C. Jogging every other day D. Using a treadmill

A. Swimming Rationale: Swimming is a particularly useful exercise to maintain joint mobility without placing pressure on the joints. Jogging, using a treadmill, and playing basketball would place pressure on the joints of the lower extremities.

After teaching the parents of a child with varicella zoster, the nurse determines that the parents have understood the teaching when they state that their child can return to school at which time? Select one: A. When the rash is completely healed B. After day 5 of the rash C. After the lesions have crusted D. Once the rash appears

C. After the lesions have crusted

A group of nursing students are reviewing information about humoral and cellular immunity. The students demonstrate understanding of this material when they identify which of the following as involved in cellular immunity? Select one: A. Antigens B. Antibodies C. T cells D. B cells

C. T cells Rationale: Cellular immunity involves T cells, which do not recognize antigens. B cells, antibodies, and antigens are involved in humoral immunity.

The nurse is developing a plan of care for an infant with heart failure who is receiving digoxin. The nurse would hold the dose of digoxin and notify the physician if the infant's apical pulse rate was: Select one: A. 120 beats per minute B. 140 beats per minute C. 100 beats per minute D. 80 beats per minute

D. 80 beats per minute Rationale: In an infant, if the apical pulse rate is less than 90 beats per minute, the dose is held and the physician should be notified.

A nurse is preparing a class for parents of infants about managing diaper dermatitis. Which of the following would the nurse include in the presentation? Select all answers that apply. Select one or more: A. Using scented diaper wipes to clean the area B. Refraining from using rubber pants over diapers C. Applying topical nystatin to the diaper area D. Washing the diaper area with an antibacterial soap E. Using a blow dryer on warm to dry the diaper area

The correct answer is: Using a blow dryer on warm to dry the diaper area, Refraining from using rubber pants over diapers Rationale: For diaper dermatitis, topical products such as ointments or creams containing vitamins A, D, and E; zinc oxide; or petrolatum help to provide a barrier. Nystatin is an antifungal agent used for diaper candidiasis. Using a blow dryer on warm to dry the area, avoiding the use of rubber pants, and using unscented diaper wipes or ones free of preservatives are appropriate. The area should be washed with a soft cloth, without harsh soaps.

A 2-month-old infant is admitted with suspected Spinal Muscular Atrophy. Which of the following findings is the nurse likely to assess? Select one or more: a. Generalized weakness b. Scoliosis c. Weak cry d. Low weight-to-length ratio

The correct answer is: Weak cry, Generalized weakness, Low weight-to-length ratio

A 5-month-old child comes to the clinic for a well-child check and catch-up immunizations. Family has low income, but child's weight and length are appropriate for age. What teaching can the nurse provide to promote good oral health? Select all that apply. Select one or more: a. Baby teeth are important for speech development and good nutrition. b. Start brushing the child's teeth every day when the first molars come in. c. Wipe the gums every day with a clean cloth or gauze. d. Baby bottle tooth decay rarely occurs in infants under 18 months of age, so the child should stop using a bottle at one year old. e. As soon as the baby's teeth come in, brush them every day.

The correct answer is: Wipe the gums every day with a clean cloth or gauze., As soon as the baby's teeth come in, brush them every day., Baby teeth are important for speech development and good nutrition. Rationale: Even if the infant doesn't have teeth yet, encourage the parent to wipe the gums with a cloth every day. Stress the importance of brushing the teeth as soon as they come in. Talk about baby bottle tooth decay and ask if the child goes to sleep with a bottle (or at the breast). Teach parents that it is important to preserve the baby teeth to promote good nutrition, appropriate speech development, and proper placement of the adult teeth. Half of children in low-income homes have not had any dental care in the past year. Many of these will have unmet dental needs.

The parents of a 10-year-old boy with cystic fibrosis restrict him from camping, sleepovers with friends, and school field trips. They time his respiratory treatments and log his diet, choosing most foods for him. They have him seen medically between scheduled appointments and call their physician frequently with concerns. The nurse considers these behaviors indicative of: a. parental depression b. need for respite care c. vulnerable child syndrome d. parental denial of the child's condition

c. vulnerable child syndrome Rationale: Descriptors fit vulnerable child syndrome. Parents "see" the child as more fragile and dependent than he actually is. Resultant protective behaviors on the part of the parents interfere with the boy's ability to grow and develop as normally as possible within the restraints of his illness.

The nurse is caring for a 1-year-old boy who was a premature infant. What must the nurse do to attain accurate developmental assessment data? a. screen with the Denver II using the child's chronological age b. use open-ended questions when discussing the child with his parent c. compare the child to his siblings d. assess for developmental progress based on the child's corrected or adjusted age

d. assess for developmental progress based on the child's corrected or adjusted age rationale: Premature infants should be compared to developmental norms using their corrected age through 3 years' chronological age. Using the child's chronological age when the screening tool is the Denver II will yield inaccurate results. Comparing the child to his siblings will not provide accurate assessment data. Open-ended questions will give parents opportunity to share comprehensive data and may be an aid to gathering better parent assessment of their premature child, but this is not the key to accurate assessment of children born prematurely.

The nurse is caring for a child who has undergone stem cell transplantation for severe combined immune deficiency. Which of the following would the nurse interpret as indicative of graft-versus-host disease? Select one: A. Chronic or recurrent diarrhea B. Maculopapular rash C. Splenomegaly D. Presence of wheezing

B. Maculopapular rash Rationale: The nurse should monitor the stem cell transplant child closely for a maculopapular rash that usually starts on the palms and soles for indication that graft-versus-host disease is developing. Wheezing and recurrent diarrhea are not typical clinical manifestations of graft-versus-host disease. Splenomegaly is associated with hypogammaglobulinemia.

The nurse assesses a child's level of consciousness, noting that the child falls asleep unless he is stimulated. The nurse documents this finding as: Select one: A. Confusion B. Obtunded C. Stupor D. Coma

B. Obtunded Rationale: Obtunded is a state in which the child has limited responses to the environment and falls asleep unless stimulation is provided. Confusion involves disorientation; the child may be alert but responds inappropriately to questions. Stupor exists when the child responds only to vigorous stimulation. Coma is a state in which the child cannot be aroused even with painful stimuli.

Which of the following findings are suggestive of congenital diaphragmatic hernia? (Select all that apply.) Select one or more: a. Barrel-shaped chest. b. Absence of breath sounds on one side. c. Current jelly stools d. Retractions and cyanosis e. Delayed passage of meconium, >48 hours

The correct answer is: Barrel-shaped chest The child with diaphragmatic hernia will prevent with severe respiratory distress, including retractions and cyanosis, shortly after birth as the abdominal contents extend into the thorax. Characteristic findings include a barrel-shaped chest and sunken abdomen. Abdominal contents may displace lungs and heart, resulting in the absence of lung sounds on one side. Absence of breath sounds on one side The child with diaphragmatic hernia will prevent with severe respiratory distress, including retractions and cyanosis, shortly after birth as the abdominal contents extend into the thorax. Characteristic findings include a barrel-shaped chest and sunken abdomen. Abdominal contents may displace lungs and heart, resulting in the absence of lung sounds on one side. Retractions and cyanosis The child with diaphragmatic hernia will prevent with severe respiratory distress, including retractions and cyanosis, shortly after birth as the abdominal contents extend into the thorax. Characteristic findings include a barrel-shaped chest and sunken abdomen. Abdominal contents may displace lungs and heart, resulting in the absence of lung sounds on one side.

Which of the following activities are appropriate for an 8-year-old child with mild Cerebral Palsy? Select one or more: a. Participating in solitary or parallel play b. Attending school c. Participating in social groups such as Girl Scouts/Boy Scouts. d. Participating in physical therapy to promote gross motor skills

b. Attending school Children with Cerebral Palsy can and should attend school. If modifications for learning are needed, the child should have an Individualized Education Plan. c. Participating in social groups such as Girl Scouts/Boy Scouts. Correct Developing social skills is a primary developmental task for a school age child. Providing opportunities to interact with same age peers is important in promoting normal growth & development for the child with CP. d. Participating in physical therapy to promote gross motor skills Gross and fine motor skills generally continue to mature in healthy children of school age. Motor skill delays are common in children with cerebral palsy so PT to promote normal development would be appropriate.

The nurse finds that an infant has stronger pulses in the upper extremities than in the lower extremities and higher blood pressure readings in the arms than in the legs. This could be indicative of which heart defect? Select one: a. Atrial septal defect b. Coarctation of the aorta c. Patent ductus arteriosus d. Transposition of the great vessels

b. Coarctation of the aorta

The written plan that the school nurse would assist in developing with the child, parents, and multidisciplinary team to foster the child's progress in school is called: a. remedial plan for education b. special education plan c. education stimulation plan d. disabled children's plan e. individualized education plan

e. Individualized education plan (IEP) rationale: The Individualized Education Plan (IEP) is required by law for each special needs student receiving services in the public schools between ages 3 years and 21 years and devised to meet preschool, elementary school, and secondary school students' individual needs. Plans carrying other names are not developed to meet this requirement.

The nurse is providing care to a child with an intussusception. The child has a bowel movement and the nurse inspects the stool. The nurse would document the stool's appearance most likely as which of the following? Select one: A. Currant jelly-like B. Greasy C. Clay-colored D. Bloody

A. Currant jelly-like Rationale: The child with intussusception often exhibits currant jelly-like stools that may or may not be positive for blood. Greasy stools are associated with celiac disease. Clay-colored stools are observed with biliary atresia. Bloody stools can be seen with several gastrointestinal disorders, such as inflammatory bowel disease.

The nurse is preparing a teaching plan for the family and their 6-year-old son who has just been diagnosed with diabetes mellitus. Which of the following would the nurse identify as the initial goal for the teaching plan? Select one: A. Educating the parents about diabetes mellitus type 1 B. Promoting independence with self-administration of insulin C. Developing a nutritionally sound, 30-day meal plan D. Developing management and decision-making skills

A. Educating the parents about diabetes mellitus type 1 Rationale: Developing basic management and decision-making skills related to the diabetes is the initial goal of the teaching plan for this child and family. The nurse would have provided a basic description of the disorder after it was diagnosed. Development of a detailed monthly meal plan would come later, perhaps after consulting with a nutritionist. It is too soon to expect the boy to administer his own insulin.

A 4-year-old boy has a history of seizures and has been started on a ketogenic diet. Which of the following food selections would be most appropriate for his lunch? Select one: A. Fried eggs, bacon, and iced tea B. A hamburger on a bun, French fries, and milk C. A grilled cheese sandwich, potato chips, and a milkshake D. Spaghetti with meatballs, garlic bread, and a cola drink

A. Fried eggs, bacon, and iced tea Rationale: The ketogenic diet involves a high intake of fats, adequate protein intake, and a very low intake of carbohydrates, resulting in a state of ketosis. The child is kept in a mild state of dehydration. Eggs and bacon are high in fat; the tea does not contain any carbohydrates. Therefore, this is the best choice. The hamburger is fat and protein, the bun is a carbohydrate, and the French fries and the milk both contain fat and protein, but both contain a lot of carbohydrates. The pasta and the sauce for the spaghetti are carbohydrates, the meatballs are protein, and the garlic bread is a carbohydrate, as is the cola drink. The grilled cheese sandwich has the fat and protein from the cheese, but the bread and chips are primarily carbohydrates, and the milkshake has fat, protein, and carbohydrates. Only the selection in A contains a ketogenic meal.

A nurse is preparing a plan of care for a child with a primary immunodeficiency. Which nursing diagnosis would the nurse most likely identify as the priority? Select one: A. Ineffective protection related to impaired humoral defenses B. Risk for delayed growth and development related to chronic illness C. Imbalanced nutrition, less than body requirements related to poor appetite D. Acute pain related to inflammatory processes

A. Ineffective protection related to impaired humoral defenses Rationale: The child with a primary immunodeficiency lacks the necessary immune responses that provide protection from infection. Therefore, the priority nursing diagnosis would be ineffective protection. Imbalanced nutrition and risk for delayed growth and development may be appropriate, but these would not be the priority. Acute pain would be more appropriate for a child with juvenile idiopathic arthritis.

When reviewing the medical record of a child, which of the following would the nurse interpret as the most sensitive indicator of intellectual disability? Select one: A. Language delay B. Vision deficit C. Preterm birth D. History of seizures

A. Language delay Rationale: Due to the extent of cognition required to understand and produce speech, the most sensitive early indicator of intellectual disability is delayed language development. A history of seizures, preterm birth, and vision deficit may be associated with intellectual disability but are not the most sensitive indicators.

A 9-year-old child has undergone a cardiac catheterization and is being prepared for discharge. The nurse is instructing the parents and child about postprocedure care. Which statement by the parents indicates that the teaching was successful? Select one: A. "He can't eat but he can drink fluids for the next 24 hours." B. "He should avoid taking a bath for about 3 days but he can shower." C. "This pressure dressing needs to stay on for 5 days from now." D. "It's normal if he says he feels like his heart skipped a beat."

B. "He should avoid taking a bath for about 3 days but he can shower." Rationale: After a cardiac catheterization, the child should avoid tub baths for about 3 days but he can shower or use sponge baths. The pressure dressing should be removed the day after the procedure and a dry sterile dressing or adhesive bandage is applied for the next several days. After the procedure, the child can resume his usual diet. Any reports of fluttering or the heart skipping a beat should be reported.

The nurse is providing care for a 14-year-old girl with severe acne. The girl expresses sadness and distress about her appearance. Which response by the nurse would be most appropriate? Select one: A. "Your condition will most likely improve in a year or two." B. "Many people feel this way; I know someone who can help." C. "If you have any scarring you can undergo dermabrasion." D. "Are you using your medicine every day?"

B. "Many people feel this way; I know someone who can help." Rationale: Depression can occur as a result of body image disturbances with severe acne. The nurse should provide emotional support to adolescents undergoing acne therapy and refer teens for counseling if necessary. Telling the girl that her condition is likely to improve in a year or two is not helpful. Asking the girl whether she uses her medicine every day or reminding her that her scars can be addressed with dermabrasion does not address her feelings of sadness and distress.

After teaching a class of nursing students about muscular dystrophy, the instructor determines that the teaching was successful when the students identify which type of muscular dystrophy as demonstrating an X-linked recessive pattern of inheritance? Select one: A. Myotonic B. Duchenne C. Limb-girdle D. Distal

B. Duchenne Rationale: Duchenne muscular dystrophy follows an X-linked recessive inheritance pattern. Limb-girdle muscular dystrophy is believed to be autosomal or X-linked inherited. Myotonic and distal muscular dystrophy follow an autosomal dominant inheritance pattern.

A child has undergone surgery using steel bar placement to correct pectus excavatum. Which of the following would the nurse instruct the parents to avoid? Select one: A. High Fowler B. Side-lying C. Semi-Fowler D. Supine Feedback

B. Side-lying Rationale: After surgery to correct pectus excavatum, the nurse would instruct the parents to avoid positioning the child on either side because this could disrupt the bar's position. Semi- or high Fowler's position and the supine position would be appropriate.

The nurse is teaching a couple about X-linked disorders. They are concerned that they might pass on hemophilia to their children. Which of the following responses indicates the need for further teaching? Select one: A. "If the mother is a carrier, her daughter could be one too." B. "The father can't be a carrier if he doesn't have hemophilia." C. "If the father doesn't have it, then his kids won't either." D. "If the mother is a carrier, her sons may have hemophilia."

C. "If the father doesn't have it, then his kids won't either." Rationale: Hemophilia is an X-linked recessive disorder. This means that both the father and the mother must have the gene for hemophilia to pass it on to their children. Also, their male children will have hemophilia, while their female children have only a 50% chance of having the disorder. If the father has hemophilia and the mother has hemophilia, their children will have the disease. If the father has hemophilia and the mother is a carrier, all their children have a 50% chance of getting the disease.

The nurse is caring for an infant with candidal diaper rash. Which topical agent would the nurse expect the physician to order? Select one: A. Retinoids B. Corticosteroids C. Antifungals D. Antibiotics

C. Antifungals Rationale: Candidal diaper rash would require a fungicide. The nurse would expect to administer topical antifungals as ordered. Corticosteroids are not typically recommended for young infants and are used for atopic dermatitis and certain types of contact dermatitis. Antibiotics would be ineffective against fungal infections. Retinoids are indicated for moderate to severe acne.

The nurse is developing a plan of care for a 7-year-old boy with diabetes insipidus. Which of the following would the nurse most likely identify as the priority nursing diagnosis? Select one: A. Deficient knowledge related to fluid intake regimen B. Imbalanced nutrition, more than body requirements related to excess weight C. Excess fluid volume related to edema D. Deficient fluid volume related to dehydration

D. Deficient fluid volume related to dehydration Rationale: The priority nursing diagnosis most likely would be deficient fluid volume related to dehydration, due to a deficiency in the secretion of antidiuretic hormone (ADH). Excess fluid would result from a disorder that leads to water retention, such as syndrome of inappropriate antidiuretic hormone (SIADH). Deficient knowledge related to fluid intake regimen is a nursing diagnosis for this child, but a secondary one. Imbalanced nutrition, more than body requirements related to excess weight would be inappropriate for this child since he probably has lost weight secondary to the fluid loss.

The nurse is caring for a special needs infant. Which intervention will be most important in helping the child reach her maximum developmental potential? Select one: A. Serving on an individualized education program committee B. Preparing a plan for her to transition to college C. Monitoring her progress in elementary school D. Directing her parents to an early intervention program

D. Directing her parents to an early intervention program Rationale: Early intervention is critical to maximizing the child's developmental potential by laying the foundation for health and development. While important, intervention in elementary or secondary school does not have the impact of early intervention. When the time arrives, it is important to have a written plan for transition to college, if this is a possibility for the grown child.

During a follow-up visit, the parents of a 5-month-old infant diagnosed with congenital heart disease tell the nurse, "We're just so tired and emotionally spent. All these tests and examinations are overwhelming. We just want to have a normal life. We're so focused on the baby that it seems like our 3-year-old is lost in the shuffle." Which nursing diagnosis would the nurse identify as most appropriate? Select one: A. Deficient knowledge related to the care of a child with congenital heart disease B. Risk for delayed growth and development related to necessary treatments C. Fear related to infant's cardiac condition and need for ongoing care D. Interrupted family processes related to demands of caring for the ill child

D. Interrupted family processes related to demands of caring for the ill child Rationale: The statements by the parents indicate that there is disruption in the family resulting from the demands of caring for the ill infant and they verbalized concern about their older child. The child may be at risk for delayed growth and development, but this is not indicated by the parents' statements. The parents may lack knowledge about their infant's condition and they may be experiencing fear about the infant's condition, but the statements reflect issues related to the family functioning.

The physician has ordered rectal diazepam (Valium) for a 2-year-old boy with status epilepticus. Which of the following instructions is essential for the nurse to teach the parents? Select one: A. Monitor for an allergic reaction to the medication. B. Watch for fever indicating infection. C. Gradually reduce the dosage as seizures stop. D. Monitor their child's level of sedation.

D. Monitor their child's level of sedation. Rationale:Diazepam is useful for home management of prolonged seizures and requires that the parents be educated on its proper administration. Monitoring the child's level of sedation is key when giving diazepam because it slows the central nervous system. Parents need to monitor the overall health of the child, including temperature when needed, but that has nothing to do with the diazepam. When the use of an anticonvulsant is stopped, gradual reduction of the dosage is necessary to prevent seizures or status epilepticus. This is not done without a physician's order. Monitoring for allergic reactions is necessary when any medications have been prescribed, but is not specific to diazepam.

A child is diagnosed with atopic dermatitis. Which laboratory test would the nurse expect the child to undergo to provide additional evidence for this condition? Select one: A. Potassium hydroxide prep B. Erythrocyte sedimentation rate C. Wound culture D. Serum immunoglobulin E (IgE) level

D. Serum immunoglobulin E (IgE) level Rationale: IgE levels are often used to evaluate for atopic dermatitis. IgE levels are elevated in this condition. Erythrocyte sedimentation rate may be used but this test is nonspecific and only indicates infection or inflammation. Potassium hydroxide prep is used to identify fungal infections. Wound culture would be done to identify a specific organism if an infection occurs with atopic dermatitis.

The nurse suspects that a 4-year-old with type 1 diabetes is experiencing hypoglycemia based on which of the following? Select all that apply. Select one or more: A. Diaphoresis B. Blurred vision C. Fruity breath odor D. Slurred speech E. Tachycardia F. Dry, flushed skin

The correct answer is: Diaphoresis, Slurred speech, Tachycardia Rationale: Manifestations of hypoglycemia include behavioral changes, confusion, slurred speech, belligerence, diaphoresis, tremors, palpitation, and tachycardia. Blurred vision; dry, flushed skin; and fruity breath odor suggest hyperglycemia.

A child is admitted with heart failure secondary to an AV canal. Which of the following should be included in the plan of care? Select one or more: a. Antibiotics for treatment of Group A strep infection b. Cluster cares c. Strict Intake & Output d. Offer small, frequent feeds e. Diuretics f. High dose aspirin therapy g. Anticipate IV access for at least 4 weeks

The correct answer is: Diuretics, Strict Intake & Output, Cluster cares, Offer small, frequent feeds.

A group of students are preparing for a class exam on skin disorders. As part of their preparation, they are reviewing information about acne vulgaris and its association with increased sebum production. The students demonstrate understanding of the information when they identify which areas as having the highest sebaceous gland activity? Select all answers that apply. Select one or more: A. Shoulders B. Upper chest C. Neck D. Face E. Back

The correct answer is: Face, Upper chest, Back

A group of nursing students are reviewing information about inflammatory bowel disease in preparation for a class discussion on the topic. The students demonstrate understanding of the material when they identify which of the following as characteristic of Crohn disease? Select all answers that apply. Select one or more: A. Loss of haustra within bowel B. Elevated erythrocyte sedimentation rate C. Distributed in a continuous fashion D. Tenesmus E. Low serum iron levels F. Most common between the ages of 10 to 20 years

The correct answer is: Most common between the ages of 10 to 20 years, Elevated erythrocyte sedimentation rate, Low serum iron levels Rationale: Crohn disease is most common between the ages of 10 and 20 years. Erythrocyte sedimentation rate is elevated and serum iron levels are low. Ulcerative colitis is distributed continuously distal to proximal, with tenesmus and loss of haustra within the bowel. Crohn disease is segmental, with disease-free skip areas common, and the bowel wall has a cobblestone appearance.

Which of the following are important nursing interventions when caring for an infant with a myelomeningocele in the preop stage? 1. Place infant prone with knees slightly flexed 2. Apply a heat lamp to facilitate drying and toughening of the sac 3. Cover the sac with a sterile dressing, using betadine to prevent infection 4. Measure head circumference daily Select one: a. 1 and 3 only b. 1 and 4 only c. 1, 3, and 4 only d. 2, 3, and 4 only e. 2 and 3 only

b. 1 and 4 only Correct Early in the preop phase (>10 hours before surgery) the child could be fed with head turned to one side.

Chronic sorrow is the periodic recurrence of grief-related feelings associated with an ongoing disparity from a loss experience. Which of the following families are at high risk for developing chronic sorrow? Select one or more: a. Parents of a child admitted with pneumonia right before Christmas b. Parents of a child who experienced pelvis and femur fractures in an automobile/bicycle accident who must spend 3 months in full spica cast. c. Parents of a child with Pierre-Robin sequence who is trach dependent d. Parents of a child with spina bifida who is wheelchair bound

c. Parents of a child with Pierre-Robin sequence who is trach dependent This child will have limitations on "normal" activities of childhood. He/she may want to be involved with other children, but be unable to participate in the same activities because he/she is dependent on technology and on specific training. What if he/she was at a friend's house and the trach was coughed out? What would the other parents do? These alterations in expectations represent ongoing periodic reminders of the loss of a healthy child. d. Parents of a child with spina bifida who is wheelchair bound This child will have some limitations on "normal" activities of childhood. He/she may want to be involved with other children, but be unable to participate in the same activities or visit friends home because of limited wheelchair access. These represent ongoing periodic reminders of the loss of a healthy child and the expected experiences that brings.

An infant has been diagnosed with Osteogenesis Imperfecta. (OI). The nurse is teaching the parents about how to care for their infant. What information is most important for the nurse to include in the instructions to the parents? Select one: a. Notify the health-care provider if your infant does not respond to sound because the infant's central nervous system fails to develop completely. b. If you note signs of infection bring your infant to the clinic because the infant has a significant immune dysfunction. c. Protect your infant from injury and handle your baby carefully because your infant's bones can break very easily d. Check the color of your infant's nail beds and mucous membranes for the signs of circulatory impairment

c. Protect your infant from injury and handle your baby carefully because your infant's bones can break very easily Rationale: OI is also known as brittle bone disease and the infant should be handled carefully and protected from injury.

A 4-month-old is brought to the Emergency Department for evaluation of constipation which began about a week ago. Mother states the child is not feeding well. On assessment, the nurse notes a weak cry, drooping eyelids, and overall floppy muscle tone. Which of the following is the priority nursing action? Select one: a. Instruct the mother not to try feeding the child until the constipation is more fully evaluated. b. Obtain stool and blood cultures and place an IV for antibiotics. c. Place a feeding tube since the child will have a decreased gag reflex. d. Ensuring that airway managment equipment, including a bag & mask and intubation supplies, is readily available.

d. Ensuring that airway managment equipment, including a bag & mask and intubation supplies, is readily available. Rationale: This child has many signs and symptoms of infantile botulism. He could lose the ability to manage his airway at an moment. These children are unstable and will require airway management at some point. This is the priority nursing intervention.

A newborn has been diagnosed with Hirschsprung's disease. The parents ask the nurse about the symptoms that lead to this diagnosis. The nurse should explain that common symptoms are: Select one: a. Currant jelly colored, gelatinous stools; pain. Incorrect. These are symptoms of intussusception. b. Projectile vomiting; altered electrolytes. c. Acute diarrhea; dehydration. d. Failure to pass meconium; abdominal distention.

d. Failure to pass meconium; abdominal distention.

The nurse is administering pain medication for a child with continuous pain from internal injuries. Which of the following methods would be ordered to dispense the medication? Select one: A. Administer the medication PRN (as needed). B. Administer the medication around the clock at timed intervals. C. Administer the mediation when pain has peaked. D. Administer the medication when the child complains of pain. Feedback

B. Administer the medication around the clock at timed intervals. Rationale: With any medication administered for pain management, the timing of administration is vital. Timing depends on the type of pain. For continuous pain, the current recommendation is to administer analgesia around the clock at scheduled intervals to achieve the necessary effect. As-needed or PRN dosing is not recommended for continuous pain. This method can lead to inadequate pain relief because of the delay before the drug reaches its peak effectiveness. For pain that can be predicted or considered temporary, such as with a procedure, analgesia is administered so that the peak action of the drug matches the time of the painful event. It is not recommended to wait until the child complains of pain because therapeutic levels will be difficult to reach at this point.

After teaching a class about the hemodynamic characteristics of congenital heart disease, the instructor determines that the teaching has been successful when the class identifies which defect as an example of a disorder involving increased pulmonary blood flow? Select one: A. Tetralogy of Fallot B. Atrial septal defect C. Hypoplastic left heart syndrome D. Transposition of the great vessels

B. Atrial septal defect Rationale: Atrial septal defect is an example of a disorder involving increased pulmonary blood flow. Tetralogy of Fallot is a defect involving decreased pulmonary blood flow. Transposition of the great vessels and hypoplastic left heart syndrome are examples of mixed disorders.

The nurse uses family-centered care to care for children in a pediatric office. Upon what concept is family-centered care based? A. The family is the constant in the child's life and the primary source of strength. B. The care provider is the constant in the child's life and the primary source of strength. C. The wishes of the family should direct the nursing care plan for the child. D. The child must be prepared to be his or her own source of strength during times of crisis.

A. The family is the constant in the child's life and the primary source of strength. Rationale: Family-centered care involves a partnership between the child, family, and health care providers in planning, providing, and evaluating care. Family-centered care enhances parents' and caregivers' confidence in their own skills and also prepares children and young adults for assuming responsibility for their own health care needs. It is based on the concept that the family is the constant in the child's life and the primary source of strength and support for the child.

The nurse is caring for a child who is receiving total parenteral nutrition (TPN) for failure to thrive. Which of the following nursing actions might the nurse take to prevent complications from this therapy? Select one: A. Use occlusive dressings and chlorhexidine-impregnated sponge (Biopatch) dressings. B. Ensure that the system remains an open system at all times. C. Adhere to clean technique when caring for the catheter and administering TPN. D. Secure all connections and open the catheter during tubing and cap changes.

A. Use occlusive dressings and chlorhexidine-impregnated sponge (Biopatch) dressings. The nurse should use occlusive dressings and chlorhexidine-impregnated sponge (Biopatch) dressings to help prevent infection. The nurse should always follow agency or institution policy and procedures, adhere to strict aseptic technique when caring for the catheter and administering TPN, ensure that the system remains a closed system at all times, and secure all connections and clamp the catheter or have the child perform the Valsalva maneuver during tubing and cap changes.

The nurse is caring for a child brought to the emergency department by a babysitter. The child needs emergency treatment and the parents cannot be contacted. What would be the nurse's best response to this situation? Select one: A. Have the babysitter sign the consent form even if she does not have signed papers to do so. B. Document failed attempts to obtain consent to allow emergency care without consent. C. Have the primary care physician for the child sign the consent form. D. Delay medical care until the child's next of kin can be contacted.

B. Document failed attempts to obtain consent to allow emergency care without consent. Rationale: Health care providers can provide emergency treatment to a child without consent if they have made reasonable attempts to contact the child's parent or legal guardian (American Academy of Pediatrics, Committee on Pediatric Emergency Medicine, 2007). If the parent is not available, then the person in charge may give consent for emergency treatment if that person has a signed form from the parent or legal guardian allowing him or her to do so. During an emergency situation, a verbal consent via the telephone may be obtained. In urgent or emergent situations, appropriate medical care never should be delayed or withheld due to an inability to obtain consent.

The parents of a 5-year-old bring their son to the emergency department because of significant eyelid edema. The mother states, "He scratched himself near his eye a couple of days ago while playing outside in the yard." The nurse suspects periorbital cellulitis based on which of the following? Select one: A. Altered visual acuity B. Evidence of discharge C. Reddened conjunctiva D. Purplish discoloration of eyelid

D. Purplish discoloration of eyelid Rationale: Periorbital cellulitis is a bacterial infection of the eyelids and tissue surrounding the eye. The bacteria may gain entry into the skin via an abrasion, laceration, insect bite, foreign body, or impetiginous lesion. It may also result from a nearby bacterial infection such as sinusitis. Findings include marked eyelid edema, purplish or red color of the eyelid, clear conjunctivae, absence of discharge, and normal visual acuity.

The nurse is caring for a previously healthy 3-month-old girl hospitalized with RSV Bronchiolitis. Which of the following treatments does the nurse expect to utilize during the course of this child's care? Select all that apply. A. Suctioning with a bulb syringe B. Oxygen via tent C. Humidification via mask or nebulizer D. Nasal suction E. Intravenous antibiotics F. Maintenance IV fluids

Nasal suction, Suctioning with a bulb syringe, Maintenance IV fluids

A nurse identifies a nursing diagnosis of impaired urinary elimination related to urinary tract infection. When developing the plan of care, which of the following would be most important for the nurse to do first? Select one: A. Develop a schedule for bladder emptying B. Encourage fluid intake C. Assess usual voiding patterns D. Monitor intake and output

The correct answer is: Assess usual voiding patterns Rationale: The first action would be to assess the child's usual voiding patterns to establish a baseline to develop an appropriate schedule for bladder emptying. Encouraging fluid intake and monitoring intake and output would be appropriate, but these would not be the first action.

The nurse is providing atraumatic care to children in a hospital setting. Which of the following are principles of this philosophy of care? Select all answers that apply. Select one or more: A. Minimize parent-child interactions B. Avoid or reduce painful procedures C. Use core primary nursing D. Minimize child control E. Provide child-centered care F. Avoid or reduce physical distress

The correct answer is: Avoid or reduce painful procedures, Avoid or reduce physical distress, Use core primary nursing Rationale: When using atraumatic care, the nurse would avoid or reduce painful procedures, avoid or reduce physical distress, use core primary nursing, maximize parent-child interactions, provide family-centered care, and provide opportunities for control, such as participating in care, attempting to normalize daily schedule, and providing direct suggestions.

A group of nursing students are reviewing information about childhood infectious diseases. The students demonstrate understanding of this information when they identify which of the following as a common childhood exanthema? Select one or more: A. West Nile virus B. Fifth disease C. Rabies D. Mumps E. Varicella

The correct answer is: Fifth disease, Varicella Rationale: Common viral exanthems include rubella, rubeola/measles, varicella, reseola, fifth disease/erythema infectiousum, hand, foot & mouth disease

he nurse is assessing a 2-day-old newborn and suspects Down syndrome based on which of the following? Select all answers that apply. Select one or more: A. Rigid joints B. Flat facial profile C. Simian crease D. Large tongue compared to mouth E. Epicanthal folds F. Downward slant to the eyes

The correct answer is: Flat facial profile, Large tongue compared to mouth, Simian crease, Epicanthal folds Rationale: Common clinical manifestations of Down syndrome include flat facial profile, upward slant to the eyes (oblique palpebral fissures), tongue that is large in comparison to the mouth size, simian, crease, epicanthal folds, and loose joints.

The nurse is examining a 15-month-old child who was able to walk at the last visit and now can no longer walk. What would be the nurse's best intervention in this case? Select one: A. Ask the parents if they have changed the child's schedule to a less active one. B. Schedule a full evaluation since this may indicate a neurologic disorder. C. Note the regression in the child's chart and recheck in another month. D. Document the findings as a developmental delay since this is a normal occurrence.

The correct answer is: Schedule a full evaluation since this may indicate a neurologic disorder. Rationale: Any child who "loses" a developmental milestone—for example, the child able to sit without support who now cannot—needs an immediate full evaluation, since this indicates a significant neurologic problem.

The nurse is preparing to administer medication to a child with a gastrostomy tube in place. Which of the following are recommended for this procedure? Select all answers that apply. Select one or more: A. Mix powdered medications well with cold water first. B. Crush tablets and mix with warm water to prevent tube occlusion. C. Mix liquid medications with a small amount of water and add directly into the tube. D. Verify medications can be crushed/opened and can be given via tube. E. Flush the tube with water after administering medications. F. Open up capsules and mix the contents with warm water.

Verify medications can be crushed/opened and can be given via tube., Crush tablets and mix with warm water to prevent tube occlusion., Open up capsules and mix the contents with warm water., Flush the tube with water after administering medications. Rationale: The correct procedure includes the following: checking proper tube placement prior to instilling medication, crushing tablets and mixing with warm water to prevent tube occlusion, opening up capsules and mixing the contents with warm water, and flushing the tube with water after administering medications. The nurse should give liquid medications directly into the tube and mix powdered medications well with warm water first.

A child is being discharged from the hospital after a 3-week stay following a motor vehicle accident. The mother expresses concern about caring for the child's wounds at home. She has demonstrated appropriate technique with medication administration and wound care. Which is the priority nursing diagnosis? Select one: a. Knowledge deficit of home care b. Altered family processes related to hospitalization c. Risk for infection related to presence of healing wounds d. Parental anxiety related to care of the child at home

d. Parental anxiety related to care of the child at home Rationale: While all of the diagnoses might have been appropriate at some point, the current focus is the mother's anxiety about caring for the child at home. The priority is to develop a plan to assist in relieving the anxiety.

A child's medical record contains the diagnosis failure to thrive (FTT). The nurse realizes: Select all that apply. a. the cause may be organic or inorganic b. it may have developmental delay as a contributing factor c. it could be related to poverty d. the growth chart shows an extended period of poor weight gain. e. that special needs children often carry this diagnosis

a, b, c, d and e rationale: All are true of failure to thrive. Physical or physiologic problems cause organic failure to thrive. Inorganic failure to thrive derives from psychosocial sources. The line between the two may not always be clear, however, since causes of the problem can be mixed.

The nurse is caring for a 15-year-old boy with chronic lung disease. Choose the appropriate interventions for this boy. Select all that apply. a. promote participation in after-school activities b. defer discussing the boy's plans for the future c. encourage parents to maintain very close supervision of their teen's activities d. urge the adolescent to make up missed schoolwork e. assist the teen in developing effective interpersonal skills f. educate school staff about the teenager's special needs

a, d, e, f Rationale: Teenagers with special healthcare needs often need to work on interpersonal skills appropriate for their developmental age. Illness, treatments, missed school days, and differences in abilities from their peers interfere with opportunities to practice relational skills with age-mates and others. Educating the school staff, promoting involvement in after-school activities, and keeping up with schoolwork assist in normalizing the teenager's development. Delay in considering the teen's future and having the parents continue close supervision do not help the adolescent move toward independence and are not suitable interventions.

A mother calls a clinic nurse to ask if her infant born prematurely should receive the seasonal influenza vaccine. The nurse's next question should be: a. "How old is your baby?" b. "Did your baby have any respiratory problems?" c. "Does your baby have any allergies?" d. "How much premature was your baby?"

a. "How old is your baby?" Rationale: Flu vaccine and all other vaccines are administered according to chronological age. Flu vaccine is recommended for all infants at 6 months of age and given yearly thereafter. An underlying respiratory problem makes flu vaccine important. Awareness of allergies is also necessary, but the first question is chronological age to determine if the infant is old enough to receive the vaccine.

A 14-year-old boy has died from drowning. When asked about organ donation, the mother refuses, saying she wants to be able to look at her boy at the funeral home. What is the best way for the nurse to respond? a. Explain that there are no visible signs of organ donation. b. Tell the parent there is great need for donated organs. c. Explain that her teen will live on in others. d. Assure the mother that the child will not suffer. e. Ask if her son ever considered organ donation.

a. Explain that there are no visible signs of organ donation. rationale: Addressing the mother's desire by assuring her that donation will not alter her son's appearance is the best response. The other statements are true, and the question about her son ever considering donation appropriate; they simply do not address the concern expressed.

The nurse is preparing a 7-year-old girl recovering from head trauma and receiving gastrostomy feedings for discharge from the hospital. Which activity is most important before the child is discharged home? a. determining the parents' ability to administer the enteral feedings b. preparing a list of home equipment and supplies needed c. assessing the parents' emotional status d. helping the family to access financial resources

a. determining the parents' ability to administer the enteral feedings Rationale: The parents' ability to maintain their child's nutrition is essential to the child's well-being. The transition can go forward while still resolving financial resource adequacy and the emotional status of the parents. Equipment and supplies will be ordered as part of discharge planning and are not needed until the parents can safely administer feedings.

Bacterial pneumonia is suspected in a 4-year-old boy with fever, headache, and chest pain. Which assessment finding would most likely indicate the need for this child to be hospitalized? Select one: a. Pale skin color b. Oxygen saturation level of 94% c. Fever d. Tachypnea with retractions

d. Tachypnea with retractions Rationale: Hospitalization would most likely be required for the child with tachypnea, significant retractions, poor oral intake, or lethargy for the administration of supplemental oxygen, intravenous hydration, and antibiotics.

Which statement by the mother of a special needs toddler requires nursing intervention? Select all that apply. a. "we give her lots of finger foods so she can feed herself" b. "she teethed late, and her teeth are still coming in so we haven't looked for a dentist." c. "we take her to the park when the weather is good" d. "she scoots around on the floor so we keep her in a playpen for safety."

b. "she teethed late, and her teeth are still coming in so we haven't looked for a dentist." d. "she scoots around on the floor so we keep her in a playpen for safety." Rationale: Safety is important, but restricting the toddler to a playpen interferes with motor development and learning. Toddlers explore their environment to develop motor skills and learn through trial and error. Being enclosed in one environment (playpen) does not provide for this. Late teething may occur in special needs children. However, dental care is essential to the health of all children, and early evaluation and care is often even more important to the special needs child. Special needs children benefit from problem prevention or correction in all arenas to support development and promote functioning at the highest possible level. Finger foods support self-feeding and the autonomy desired by toddlers. Excursions to the park provide motor and sensory stimulation and fun.

The nurse is providing education to the parents of a 2-year-old boy with hydrocephalus who has just had a ventriculoperitoneal shunt placed. Which information is most important for the parents to be taught? a. "limit the amount of t.v. he watches" b. "watch for changes in his behavior or eating patterns" c. "call the doctor if he gets a headache." d. "always keep his head raised 30 degrees"

b. "watch for changes in his behavior or eating patterns" rationale: Changes in behavior or in eating patterns can suggest a problem with his shunt, such as infection or blockage. Irritability, lack of appetite, increased crying, or inability to settle down may indicate increased intracranial pressure. Any headache needs to be monitored, but if it goes away quickly, such as after eating, it probably isn't a problem. It is not necessary to keep the child's head raised 30 degrees. The child's shunt will not be affected by the amount of television viewed.

Media is beginning to promote immunizations for the upcoming influenza season. The mother of a premature infant 7 months chronological age and 5 months corrected age asks about immunizing her child. The nurse responds: a. "no child receives seasonal flu vaccine until age 4 years" b. "your child as a 7-month-old should be immunized." c. "the child's corrected age makes him too young." d. "premature infants should not be immunized against the seasonal flu until 1 year old chronologically."

b. "your child as a 7-month-old should be immunized." Rationale: Infants receive influenza vaccine at 6 months of age. Chronological age is used for premature infants. All the other responses are incorrect.

The mother of a 10-year-old being treated for kidney failure speaks very broken English and is clearly overstressed. What is the priority nursing intervention? a. assuring her and demonstrating that the child will be well cared for b. gaining more information about her stress c. encouraging her to go home and get some rest d. providing her with a bed and food in the child's room

b. gaining more information about her stress rationale: The priority intervention is to determine the sources of the mother's stressors. What are her fears and concerns? What pressures are present in her life? An interpreter may be necessary to ensure effective communication. Until then, the nurse cannot be sure that the other measures are appropriate, although they are caring.

A newborn presents with gastroschisis. A single stage repair is scheduled for tomorrow morning. The nurse knows that a priority nursing intervention for this child is: Select one: a. Ensure adequate calories are given before the child is made NPO. b. Begin teaching the parents about infant ostomy care. c. Administer IV fluids and monitor intake and output. d. Elevate head of bed to minimize risk of aspiration.

c. Administer IV fluids and monitor intake and output. Rationale: The child is at risk for fluid loss from the exposed bowel and the need to be in a warmer or isolette. Since the child is NPO preoperatively, fluid volume balance is maintained with intravenous fluids and monitoring of I&O.

Which of the following is the most accurate regarding the structure and function of the infant of child's respiratory system? Select one: a. The respiratory tract in the child is fully developed by age 2 b. Infants and young children have smaller tongues in proportion to their mouths. c. Most infants are nasal breathers rather than mouth breathers d. The diameter of the child's trachea is the same as that of adults.

c. Most infants are nasal breathers rather than mouth breathers

A 3-month-old boy was diagnosed with failure to thrive. What action will be most helpful in assisting the nurse to determine if there is an inorganic cause? a. reviewing the medical records for a history of prematurity or a congenital anomaly b. assessing for adequate calorie intake through recording ounces of formula consumed c. observing the mother-child interaction during feeding and hygiene activities d. observing the child's interest in and ability to feed

c. observing the mother-child interaction during feeding and hygiene activities Rationale: Observing the mother:child interaction during feeding and hygiene activities would disclose lack of knowledge of child care, poor feeding techniques, or inappropriate maternal bonding and interaction as inorganic causes or failure to thrive. The child's lack of interest in or inability to feed would indicate organic causes, as would determining that the child consumed adequate calories for age and finding a history of prematurity or congenital anomaly.

The parents of a 9-year-old girl who is dying from cancer are distraught and guilt-ridden when they find that treatment is no longer successful. What is the best way for the nurse to respond? a. explain that it is not fair to the child to continue present treatment. b. tell the parents there is no more that can be done. c. ask the parents if they wish to fill out a do-not-resuscitate order. d. assure the parents that expert care of their child will continue.

d. assure the parents that expert care of their child will continue. Rationale: The nurse needs to make sure the parents know that the child is not being abandoned by the healthcare team. Instead, treatment is changing but not ending. It is not true there is not anymore that can be done for the child. Palliative care can relieve symptoms and provide comfort even though it will not cure. Waiting to inquire about a do-not-resuscitate decision until parents have some adjustment time is considerate. Claiming it is not fair to continue treatment may enhance the guilt already being expressed by the parents.

Nurses explain that before the parents of a premature infant leave the hospital with their baby, the child must: a. be immunized against pertussis b. have no apnea episodes c. be able to nipple feed d. maintain oxygenation in a car seat

d. maintain oxygenation in a car seat Rationale: Maintaining satisfactory oxygenation saturation while sitting in a car seat is necessary prior to hospital discharge for premature infants. Special padding of the seat may be necessary. All immunizations will be given based on chronological age with pertussis first given at 2 months. Premature infants may be discharged from hospital units with feeding tubes and apnea monitors.

A 15-year-old male had a tonsillectomy 10 days ago. Earlier in the day, he complained that his throat was very sore and it felt "like something was stuck there." His mother calls to report that he just vomited "quite a bit" of bright red blood. Which of the following is the most appropriate response by the nurse? Select one: a. "Can you bring him to the clinic so we can check it? We'll squeeze him in anytime." b. "It's very common to vomit once or twice after tonsillectomy. Watch him and let us know in the morning how he is feeling." c. "When did he last take pain medication and is he having any pain right now?" d. "Have him wear an ice collar for 20 minutes. That should help decrease any swelling or bleeding." e. "Go to the nearest emergency room right away."

e. "Go to the nearest emergency room right away." Rationale: A post-tonsillectomy bleed can be life-threatening if not appropriately controlled. The most appropriate response is to tell the mother to take the child to the closest emergency room. It is common to vomit once or twice after tonsillectomy, but that would normally be in the first 24 hours post-op. An ice collar will help with pain and swelling, but not with bleeding. At some point, we need to know about when the last pain med was taken, but this is not the priority in the presence of a bleed. A clinic does not have the capability to correct the bleeding, so it is not the best response.


संबंधित स्टडी सेट्स

Unit 6: Life Insurance Policy Options

View Set

LifePac History 1001 Test study guide (Ancient Civilizations)

View Set

Chapter 11 Networked Applications

View Set

ISDS final: the internet, web, and electronic commerce

View Set

Chapter 8: Assessing a New Venture's Financial Strength and Viability

View Set